GSM 143 Fa Solutions

You might also like

Download as pdf or txt
Download as pdf or txt
You are on page 1of 72

Lecture Notes on Functional Analysis

Review of Notation and Solutions to Homework Problems

Alberto Bressan

Review of main notation

R the field of real numbers.


C the field of complex numbers.
K a field of numbers, either R or C.
Re z, Im z the real and imaginary part of a complex number z.
z̄ = a − ib the complex conjugate of the number z = a + ib ∈ C.
[a, b] a closed interval, ]a, b[ an open interval, ]a, b], [a, b[ half-open intervals.
Rn the n-dimensional Euclidean space.
h·, ·iscalar product on the Euclidean space Rn .
. p
|v| = (v, v) the Euclidean length of a vector v ∈ Rn .
.
A \ B = {x ∈ A , x ∈ / B} a set-theoretic difference.
A the closure of a set A.
∂A the boundary of a set A.

1 if x ∈ A ,
χ the indicator function of a set A. χ (x) =
A A 0 if x ∈
/ A.
f : A 7→ B a mapping from a set A into a set B,
a 7→ b = f (a) the function f maps the element a ∈ A to the element b ∈ B.
.
= equal, by definition.
⇐⇒ if and only if
C(E) = C(E, R) vector space of all continuous, real valued functions on the metric space E.
C(E, C) vector space of all continuous, complex valued functions on the metric space E.
BC(E) space of all bounded, continuous, real valued functions f : E 7→ R, with norm
kf k = supx∈E |f (x)|.

1
`1 , `p , `∞ spaces of sequences of real (or complex) numbers.
L1 (Ω), Lp (Ω), L∞ (Ω) Lebesgue spaces.
W k,p (Ω) Sobolev space of functions whose weak partial derivatives up to order k lie in Lp (Ω),
for some open set Ω ⊆ Rn .
H k (Ω) = W k,2 (Ω) Hilbert-Sobolev space.
C k,γ (Ω) Hölder space of functions u : Ω 7→ R whose derivatives up to order k are Hölder
continuous with exponent γ ∈ ]0, 1].
k · k = k · kX norm on a vector space X.
(·, ·) = (·, ·)H inner product on a Hilbert space H.
X ∗ is the dual space of X, i.e. the space of all continuous linear functionals x∗ : X 7→ K.
hx∗ , xi = x∗ (x) duality product of x∗ ∈ X ∗ and x ∈ X.
xn → x strong convergence in norm; this means kxn − xk → 0.
xn * x weak convergence.

ϕn * ϕ weak-star convergence.
f ∗g convolution of two functions f, g : Rn 7→ R.
∇u = (ux1 , ux2 , . . . , uxn ) gradient of a function u : Rn 7→ R.
 α1  α2  αn
∂ ∂ ∂
Dα = ∂x1 ∂x2 · · · ∂xn = ∂xα11 ∂xα22 · · · ∂xαnn a partial differential operator of
.
order |α| = α1 + α2 + · · · + αn .
meas(Ω) the Lebesgue measure of a set Ω ⊂ Rn .
Z Z
. 1
− f dx = f dx the average value of f over the set Ω.
Ω meas(Ω) Ω

2
Solutions to Homework Problems

Chapter 2

1. (i) It is not a normed space. (N2) fails when λ < 0.


 
(ii) It is a normed space but not complete. The sequence xn = 1, 12 , 31 , . . . , n1 , 0, 0, . . . is
Cauchy but does not have a limit in X.
. P n
(iii) It is a normed space but not complete. The sequence pn (x) = nk=0 xn! is Cauchy, but
does not converge to any element of X. Indeed, its uniform limit on the interval [0, 1] is the
function f (x) = ex , which is not a polynomial.
(iv) It is a normed space of dimension 3. It is also a Banach space. Every finite dimensional
normed space is complete.
(v) It is a normed space, but not complete. Indeed, consider the functions
 h i
1
 h i


 0 if x ∈ 0, 2 ,
1

 0 if x ∈ 0, 2 ,
 

 
. .
   h i
1 1 1 1
f (x) = i i fn (x) = n x − 2 if x ∈ 2 , 2 + n ,
 1

1 if x ∈ 2 , 1 ,

 


 h i

1 if x ∈ 21 + n1 , 1 .

Then the sequence of continuous functions (fn )n≥1 is a Cauchy sequence without any limit in
1
the space X. Namely, kf − fn kL1 ([0,1]) = 2n → 0 as n → ∞, but f ∈
/ X.
(vi) For p < 1 this is not a norm, because (N3) fails. For example, in R2 take x = (1, 0),
y = (0, 1). Then kxk = kyk = 1, but kx + yk = 21/p > 2.

2. (N1) k(x, y)k = max{kxk, kyk} ≥ 0.


k(x, y)k = 0 if and only if max{kxk, kyk} = 0 if and only if kxk = kyk = 0 if and only if
x = y = 0.
(N2) k(λx, λy)k = max{kλxk, kλyk} = max{|λ| kxk, |λ| kyk} = |λ| max{kλxk, kλyk} =
|λ| k(x, y)k.
(N3) k(x+x̃, y+ỹ)k = max{kx+x̃k, ky+ỹk} ≤ max{kxk+kx̃k , kyk+kỹk} ≤ max{kxk, kyk}+
max{kx̃k, kỹk} = k(x, y)k + k(x̃, ỹ)k.
To prove that X × Y is a Banach space, let (xn , yn )n≥1 be a Cauchy sequence in X × Y . Then
(xn )n≥1 is a Cauchy sequence in X and (yn )n≥1 is a Cauchy sequence in Y . Since X, Y are
complete, there exist the limits xn → x, yn → Y . This implies (xn , yn ) → (x, y), showing that
X × Y is complete as well.

3
3. (i) Let (x̄, ȳ) ∈ X × X and ε > 0 be given. Choose δ = ε/2. If k(x, y) − (x̄, ȳ)k ≤ δ then

k(x + y) − (x̄ + ȳ)k ≤ kx − x̄k + ky − ȳk ≤ δ + δ = ε.

This proves that the map (x, y) 7→ x + y is continuous at the point (x̄, ȳ).
(ii) Let α ∈ K, x ∈ X, and ε > 0 and be given. Choose
 
. ε/2 ε/2
δ = min 1, , .
1 + kxk 1 + |α|
Assume |β − α| ≤ δ, ky − xk ≤ δ. Then kyk ≤ 1 + kxk and
ε ε
kβy − αxk ≤ kβy − αyk + kαy − αxk ≤ |β − α| (1 + kxk) + |α| ky − xk ≤ + = ε.
2 2

4. If V is a subspace of X it is clear that the properties (N1)–(N3) of the norm remain valid.
Now assume that X is a Banach space and that V is a closed subset of X. If (vn )n≥1 is a
Cauchy sequence in V , then it is also a Cauchy sequence in X. Since X is complete, one has
the convergence xn → x for some point x ∈ X. Since V is closed, x ∈ V , proving that V is
complete as well.

P
5. Let X be a Banach space and assume that n≥1 kxn k < ∞. Consider the sequence of
. Pn
partial sums yn = k=1 xk . This is a Cauchy sequence. Indeed, for m < n we have


X X
kym − yn k = xk
≤ kxk k → 0 as m, n → ∞.
m<k≤n m<k≤n

Since X complete, this sequence has a limit: yn → y = ∞


P
k=1 xk .

Viceversa, assume that every absolutely convergent series of elements of X has a sum. Let
(yn )n≥1 be a Cauchy sequence of elements in X. For each integer k ≥ 1, choose an index Nk
such that
kym − yn k ≤ 2−k whenever m, n ≥ Nk .
P
It is not restrictive to assume N1 < N2 < N3 < · · · . Consider the series k xk , where

x1 = yN1 , x2 = yN2 − yN1 , ··· xk = yNk − Nk−1 , ···


1−k for all k ≥ 2. By assumption, there exist
P convergent, because kxk k ≤ 2
This is absolutely
the sum y = k xk . We claim that the sequence (yn )n≥1 converges to y. Indeed, let ε > 0 be
given and choose k large enough so that 22−k < ε. If n ≥ Nk then
X
kyn − yk ≤ ky − yNk k + kyNk − yk ≤ 2−k + kxν k ≤ 2−k + 21−k < ε.
ν≥Nk

6. (i) Let x0 , x00 ∈ coA, and assume λ ∈ [0, 1]. We need to prove that x = λx0 + (1 − λ)x00 ∈ A.
By assumption we have
N 0 N00
X X
0
x = θk0 a0k , 00
x = θk00 a00k .
k=1 k=1

4
Choosing

λθk0 if k = 1, . . . , N 0 ,

0 00
N =N +N , θk = 00
(1 − λ)θk−N 0 if k = N 0 + 1, . . . , N 0 + N 00 ,

we have the representation x = N


P
k=1 θk ak ∈ coA.

(ii) Let x = N
P
k=1 θk ak ∈ coA and let K be any convex set containing A. Then K contains
all elements a1 , . . . , aN . Being convex, K must also contain all convex combinations of these
elements. In particular, x ∈ K.
The converse inclusion is trivial: since coA is a convex set containing A, the intersection of all
convex sets that contain A cannot be larger than coA.

7. (i) Let A be open and consider any point x = N


P
k=1 θk ak ∈ coA. We can assume θ1 > 0.
Since a1 ∈ A and A is open, there exists a radius r > 0 such that B(a1 , r) ⊂ A. Then

n N
X o
B(x, θ1 r) = θ1 y + θk ak ; y ∈ B(a1 , r) ⊂ coA
k=2

This shows that x lies in the interior of coA, hence coA is open.
(ii) Let A be bounded, say A ⊆ B(0, R) for some radius R > 0. Then coA ⊆ B(0, R) because
B(0, R) is a bounded convex set containing A. Hence coA is bounded.
(iii) If A ⊆ B(0, r1 ) and B ⊆ B(0, r2 ), then A + B ⊆ B(0, r1 + r2 ), showing that A + B is
bounded.
(iv) Let (xn )n≥1 be a sequence of points in A + B, with xn → x as n → ∞. We need to prove
that x ∈ A + B. By assumption xn = an + bn , for some an ∈ A and bn ∈ B. Since B is
compact, by possibly choosing a subsequence and relabeling we can assume bn → b for some
b ∈ B. We now have
 
lim sup kam − an k = lim sup kxm − bm − (xn − bn )k ≤ lim sup kxm − xn k + kbm − bn k = 0.
m,n→∞ m,n→∞ m,n→∞

Indeed, the sequences (xn )n≥1 and (bn )n≥1 are both convergent, hence they are Cauchy se-
quences. This shows that the sequence (an )n≥1 is Cauchy as well. Hence there exists the limit
an → a for some a ∈ A, because X is a Banach space and A ⊆ X is closed. We conclude that
x = a + b ∈ A + B, proving that the sum A + B is closed.
(v) In R2 , consider the two closed subsets
. .
A = {(x, y) ; x > 0, y > 0, xy ≥ 1} , B = {(x, y) ; x < 0, y > 0, xy ≤ −1} .

Then the set 


A+B = (x, y) ; y > 0
is open, not closed.
(vi) The inclusion A + A ⊆ 2A is always trivially true. On the other hand, if A is convex,
0 a+a0
then any element a + a0 ∈ A + A can be written as a+a
2 + 2 ∈ 2A. Hence A + A ⊆ 2A.

5
(vii) Assume that A is closed and A + A = 2A. Let a, b ∈ A. By assumption there exists
c ∈ A such that a + b = 2c. This implies a+b
2 ∈ A. Repeating the argument, we obtain that

1 a+b 1 1 3 1 1 a+b 3 1
· + · b = a + b ∈ A, ·a+ · = a + b ∈ A.
2 2 2 4 4 2 2 2 4 4
By induction, we obtain
 
k k
a+ 1− n b ∈ A for all k = 0, 1, . . . 2n . (1)
2n 2

Since the set of all convex combinations of the form (1) are dense in the set of all convex
combinations, and we are assuming that A is closed, we conclude that A is convex.
Notice that the result would fail if A were not closed. For example, let A be the set of all
rational numbers contained inside [0, 1].

8. (i) Let S be convex. Let x, y ∈ S and let θ ∈ [0, 1]. We need to show that θx + (1 − θ)y ∈ S.
By assumption, there exist sequences xn → x, yn → y, with xn , yn ∈ S for every n ≥ 1. By
convexity, θxn + (1 − θ)yn ∈ S. We now have the convergence θxn + (1 − θ)yn → θx + (1 − θ)y,
showing that S is convex.
(ii) Let S be symmetric and assume x ∈ S. Then there exists a sequence xn → x , with xn ∈ S
for every n ≥ 1. Therefore −xn ∈ S and −xn → −x. This shows that −x ∈ S, hence S is
symmetric.

9. (i) Let S be convex. Assume y1 , y2 ∈ Λ(S) and θ ∈ [0, 1]. Then there exist x1 , x2 ∈ S such
that y1 = Λ(x1 ), y2 = Λx2 . Since Λ is linear, this implies

θy1 + (1 − θ)y2 = θΛ(x1 ) + (1 − θ)Λ(x2 ) = Λ(θx1 + (1 − θ)x2 ) ∈ Λ(S),

because S is convex.
(ii) Let S be symmetric. Assume y ∈ Λ(S). Then y = Λ(x) for some x ∈ S. This implies
−y = Λ(−x) ∈ Λ(S) because S is symmetric.

10. If Λ is not continuous, then it is not bounded. Hence one can find a sequence of points
.
xn ∈ X with kyn k ≤ 1 but kΛ(yn )k ≥ n for every n ≥ 1. Defining the sequence xn = n−1/2 yn
we have xn → 0 but kΛ(xn )k ≥ n1/2 → ∞, against the assumption.

11. (i) If S is a finite set, then span(S) is a finite dimensional space. By Theorem 2.20, it is
complete. Being complete, it must be a closed subset of X.
.
(ii) Since X is a Banach space, the closure Y = span(S) is complete, hence it is a Banach space
as well. For each n ≥ 1, consider the finite dimensional subspace Vn = span{v1 , . . . , vn } ⊂ Y .
Being finite dimensional, this subspace is complete, hence it must be closed. It is easy to
see that Vn has empty interior. Indeed, since by assumptions the vectors {v1 , . . . , vN +1 } are
1
linearly independent, for every y ∈ Vn the sequence y + m vn+1 does not lie in Vn and converges
to y as m → ∞.

6
According to Baire’s category theorem, the complement
[ \
Y \ Vn = (Y \ Vn )
n≥1 n≥1

is everywhere dense in Y . Observing that span(S) = ∪n≥1 Vn , this shows that span(S) 6= Y =
span(S).

12. If x, y ∈ X and α ∈ R, then


Re(Φ(x + y)) = Re(Φ(x)) + Re(Φ(y)) , Re(Φ(αx)) = αRe(λ(x)) .
Moreover, Im(Φ(x)) = Re(−iΦ(x)) = − Re(Φ(ix)). Hence
Φ(x) = Re(Φ(x)) + Im(Φ(x)) = φ(x) − iφ(ix) .

13. If X is finite dimensional, let {e1 , . . . , en } be a basis. Then every linear functional is
continuous and is a linear combination of the n linear functionals e∗1 , . . . , e∗n where e∗i (ej ) = δij .
On the other hand, if X is infinite dimensional, let {e1 , e2 , . . .} be a countable family of
linearly independent vectors. For each n, consider the subspace Vn = span{e1 , . . . , en }. Let
ϕn : Vn 7→ R be the linear functional such that
ϕn (en ) = 1 , ϕn (ek ) = 0 for all k < n .
Since Vn is finite dimensional, ϕn is continuous. By Hahn-Banach, it can be extended to a
continuous linear functional ϕn : X 7→ R.
It remains to check that the countable set of functionals {ϕn ; n ≥ 1} is linearly independent.
If
N
X
ak ϕk (x) = 0 for all x ∈ X ,
k=1
let j be the smallest index such that aj 6= 0. Then
N
X
0 = ak ϕk (ej ) = aj ,
k=1

reaching a contradiction.

14. If x = (x1 , x2 , . . .) ∈ `p , then


 1/p ∞
X 1/p
kxk`∞ = sup |xn | = sup |xn |p ≤ |xn |p = kxk`p .
n n
n=1

Hence x ∈ `∞
and the embedding `p ⊆ `∞ is bounded, with operator norm 1. Moreover, if
p < q < ∞, then

!1/q ∞
!1/q ∞
!1/q
kxkq−p
X X X
q p q−p p
kxk`q = |xn | = |xn | |xn | ≤ |xn | · `∞
n=1 n=1 n=1

p/q (q−p)/q p/q (q−p)/q


= kxk`p · kxk`∞ ≤ kxk`p · kxk`p = kxk`p .

7
15. Consider first the case 1 ≤ p < ∞. Let x = (x1 , x2 , . . .) ∈ `p . Define the sequence of
linear combinations
n
. X .
yn = (x1 , x2 , . . . , xn , 0, 0, . . .) = xk ek ∈ V = span{ek ; k ≥ 1} . (2)
k=1

Then
kx − yn kp`p =
X
|xk |p → 0 as n → ∞ .
k>n

Next, assume x = (x1 , x2 , . . .) ∈ `∞ . Define the sequence of approximations yn as in (2). If


x ∈ c0 , i.e. if limk→∞ xk = 0, then

kx − yn k`∞ = sup |xk | → 0 as n → ∞ .


k>n
.
Hence V = span{ek ; k ≥ 1} is dense on c0 . To prove that c0 coincides with the closure of V , it
remains to prove that c0 is closed. Consider a sequence of elements xm = (xm,1 , xm,2 , xm,3 , . . .),
with xm ∈ c0 for every m ≥. Assume that limm→∞ kxm − xk`∞ = 0. We claim that x ∈ c0
as well. Indeed, let ε > 0 be given. By assumption, there exists an integer m large enough so
that kxm − xk`∞ < ε/2. Since xm ∈ c0 , there exists N large enough so that |xm,n | < ε/2 for
all n > N . Hence
ε ε
|xn | ≤ |xn − xm,n | + |xm,n | ≤ kx − xm k`∞ + |xm,n | < + = ε.
2 2
Since ε > 0 was arbitrary, this proves limk→∞ xk = 0.

16. To fix the ideas, we assume 1 ≤ p < ∞. The case p = ∞ is similar.


.
(i) If M = supk |λk |, then
!1/p !1/p
X X
kΛxk`p = |λk xk |p ≤ M· |xk |p = M kxk`p .
k k

This shows that kΛk ≤ M . On the other hand, given any ε > 0, choose k such that |λk | >
M − ε. Then
kΛek k`p = kλk ek k`p = |λk |kek k`p ≥ (M − ε) kek k`p .
This shows that kΛk ≥ M − ε. Since ε > 0 was arbitrary, we conclude that kΛk = M .
To prove (ii), assume that the sequence (λk )k≥1 is unbounded. Then

kΛk ≥ sup kΛek k`p ≥ sup |λk | = ∞ .


k≥1 k≥1

P∞ −k e .
In this case, we can extract a subsequence with |λnk | ≥ 3k . Consider the vector v = k=1 2 nk
Observe that this sum is well defined, because the series is absolutely convergent:
X X
k2−k enk k`p = |2−k | = 1 .
k≥1 k≥1

8
We now have X
Λv = λnk 2−k enk ∈
/ `p
k

Indeed,
m
X m
X
−k
lim |λnk 2 | ≥ lim (3/2)k = ∞ .
m→∞ m→∞
k=1 k=1

17. To fix the ideas, assume 1 ≤ p < ∞.


(i) Let g ∈ L∞ (Ω), say with kgkL∞ = K. If f ∈ Lp (Ω), then
Z 1/p Z 1/p
kf gkLp = |f g|p dx ≤ K p |f |p dx ≤ Kkf kLp .
Ω Ω

Hence the norm of the multiplication operator satisfies kMg k ≤ K. On the other hand, for
any ε > 0 there exists a bounded measurable subset A ⊆ Ω with strictly positive measure
such that |g(x)| ≥ K − ε for all x ∈ A. Let f = χ be the characteristic function of the set
A
A. Namely, f (x) = 1 if x ∈ A and f (x) = 0 if x ∈
/ A. Then
Z 1/p Z 1/p
p p
kMg f kLp = kf gkLp = |g| dx ≥ (K − ε) dx
A A

 1/p
= (K − ε) · meas(A) = (K − ε) · kf kLp .

This proves that kMg k ≥ K − ε.


(ii) Next, assume g ∈/ L∞ . Then for every constant K ≥ 1 we can find a bounded measurable
set A with strictly positive measure such that |g(x)| ≥ K − ε for all x ∈ A. Arguing as before,
taking f = χ we conclude that kMg k ≥ K. Since K is now arbitrary, we conclude that the
A
operator Mg is unbounded.
To obtain a function f ∈ Lp such that f g ∈
/ Lp , we first construct a sequence of disjoint,
bounded, measurable subsets An ⊂ Ω with the following properties.

1) Each An has strictly positive measure.

2) |g(x)| ≥ 3n for every x ∈ An .

/ L∞ (Ω \ ∪nj=1 Aj ).
3) For every n ≥ 1 one has g ∈

This sequence of subsets can be constructed by induction on n. Taking


∞ χ
. X −n An
f (x) = 2
kχ kLp
n=1 An

we have that f ∈ Lp (Ω) but f g ∈


/ Lp (Ω).

9
18. For any given a < b, the map
Z b
.
f →
7 ϕ(f ) = f (x) dx
a

is a bounded linear functional on Lp (R). By definition, the weak convergence fn * f implies


ϕ(fn ) → ϕ(f ).

19. (i) A straightforward computation yields


1 1−p
kfn kL∞ = , kfn kLp = n p .
n
As n → ∞, for every p > 1 the Lp norm of fn approaches zero.
(ii) On the other hand, kfn kL1 = 1 for every n.
Consider any subsequence, say fnk , with n1 < n2 < n3 < · · · By choosing a further subse-
quence and relabeling, it is not restrictive to assume nk+1 > 3nk .
Define the function g ∈ L∞ (R) by setting
g(x) = 0 if n2k−1 < x ≤ n2k
g(x) = 1 if n2k < x ≤ n2k+1 .
.
Then the map f 7→ ϕ(f ) = f g dx is a bounded linear functional linear on L1 (R). The above
R

definition implies
2 1
ϕ(fn2k ) ≥ , ϕ(fn2k+1 ) ≤ .
3 3
Therefore the subsequence ϕ(fnk ) has no limit. As a consequence, the original subsequence
does not converge weakly.

20. For every y ∈ Y , let Λy = (Λ1 y, Λ2 y, . . . , Λn y) ∈ Rn . By the Hahn-Banach extension


theorem, each linear functional Λj : Y 7→ R can be extended to a bounded linear functional
e j : X 7→ R with norm kΛ
Λ e j k = kΛj k ≤ kΛk. For x ∈ X, define the extension Λx
e =

e 1 x, Λ e n x) ∈ Rn . This is a bounded linear operator, with norm kΛk
e 2 x, . . . , Λ e ≤ n1/2 kΛk.
Indeed, if kxk ≤ 1, then
 1/2
X n  1/2
|Λx|
e =  e j x|2 
|Λ ≤ nkΛk2 = n1/2 kΛk .
j=1

21. Let φ(x1 , x2 ) = ax1 + bx2 . To prove (i) we write


.
kφk∞ = sup |ax1 + bx2 | ≤ sup (|ax1 | + |bx2 |) ≤ max{|a|, |b|}.
|x1 |+|x2 |≤1 |x1 |+|x2 |≤1

Choosing  a 
 ,0 if |a| ≥ |b| ,
 |a|


(x̄1 , x̄2 ) =
  b
if |a| ≤ |b| ,

 0,

|b|

10
one obtains the converse inequality

kφk∞ ≥ |ax̄1 + bx̄2 | = max{|a| , |b|} .

To prove (ii) we write


.
kφk∞ = sup |ax1 + bx2 | ≤ sup (|ax1 | + |bx2 |) ≤ |a| + |b| .
|x1 |≤1, |x2 |≤1 |x1 |≤1, |x2 |≤1

Choosing
a b
(x̄1 , x̄2 ) = , ,
|a| |b|
one obtains the converse inequality

kφk∞ ≥ |ax̄1 + bx̄2 | = |a| + |b| .

Using the discrete Hölder inequality, (iii) is proved by


. 1
kφkp = sup |ax1 + bx2 | ≤ sup (|ax1 | + |bx2 |) ≤ (|a|p + |b|p ) p .
1 1
(|x1 |q +|x2 |q ) q ≤1 (|x1 |q +|x2 |q ) q ≤1

Choosing
  1−q  
q
(x̄1 , x̄2 ) = |a|q + |b|q · |a|q−1 sign a , |b|q−1 sign b ,
one obtains the converse inequality
1
kφkp ≥ |ax̄1 + bx̄2 | = (|a|p + |b|p ) p .

22. By assumption, B ⊂ X is a convex set with the following property. For every non-zero
vector x ∈ X, there exists ηx > 0 such that

B ∩ {t x ; t ∈ K} = {t x ; |t| ≤ ηx } . (3)

This clearly implies B = −B and 0 ∈ B. Moreover, the functional




 0 if x = 0 ,
.

kxk = min {r ≥ 0 ; x ∈ rB} = (4)
1
6 0,
if x =


ηx

is well defined and satisfies

kxk ≤ 1 if and only if x∈B.

It remains to prove the properties (N1)–(N3) of a norm. From (3)-(4) one immediately obtains

kxk = 0 if and only if x = 0,

λαx ∈ B ⇐⇒ |λ| |α| ≤ ηx ⇐⇒ |α| ≤ θλx = θ|λ|x .

11
Hence
ηx
θ|λ|x = , kλxk = |λ| kxk .
|λ|

To prove the sub-additivity of the norm, consider any x, y 6= 0. Observe that

x ∈ kxk B , y ∈ kyk B .

Since B is a convex set containing the origin, this implies

x + y ∈ (kxk + kyk)B ,
.
kx + yk = min{r ; x + y ∈ rB} ≤ kxk + kyk .

23. (i) =⇒ (iii) Let (fj )j≥1 be a Cauchy sequence for the distance d. Let K ⊂ Ω be any
compact subset and let ε > 0 be given. Since the open sets Aj cover K, there exists N large
enough such that K ⊂ N
S
k=1 Ak . This implies
 
lim sup sup |fj (x) − f` (x)| ≤ 2N lim d(fj , f` ) = 0.
j,`→∞ x∈K j,`→∞

Therefore, the sequence (fj )j≥1 converges uniformly on the compact set K.
An identical argument shows that (ii) =⇒ (iii).
(iii)=⇒ (ii) Assume that the sequence (fj )j≥1 converges uniformly on compact subsets of Ω.
.
Let ε > 0 be given. Choose N so that 2−N < ε. Then the set K = ∪1≤k≤N Ak is a compact
subset of Ω. We thus have the estimate
N ∞
X pk (fj − f` ) X pk (fj − f` )
lim sup d(fj , f` ) = lim sup 2−k + lim sup 2−k
j,`→∞ j,`→∞ k=1 1 + pk (fj − f` ) j,`→∞ 1 + pk (fj − f` )
k=N +1

≤ 0 + 2−N < ε .

Hence the sequence (fj )j≥1 is Cauchy w.r.t. the distance d(·, ·).
An identical argument shows that (iii) =⇒ (ii).

24. (i) There is no guarantee that the series ∞ −k


P
k=1 2 pk (f ) converges. For example, take
Ω = ]0, 1[ and define
 h 1 k − 1 i
.
pk (f ) = max |f (x)| ; x ∈ Ak = , .
k k

If f (x) = e1/x then



X ∞
X
−k
2 pk (f ) = 2−k ek = ∞ .
k=1 k=1

(ii) Choose N large enough so that 2−N < ε. Then, if p1 (f ) = · · · = pN (f ) = 0, then for any
λ > 0 we have
∞ ∞
X
−n pn (λ−1 f ) X pn (λ−1 f )
2 ≤ 2−n < 2−N < ε .
1 + pn (λ−1 f ) 1 + pn (λ−1 f )
n=1 n=N +1

12
Hence λ−1 f ∈ Bε for every λ > 0. By definition, this implies kf kε = 0, showing that the
property (N1) of the norm can fail.

25. Assume x ∈ span(S), so that


n
X
x = αk xk , αk ∈ K, xk ∈ S .
k=1
P
Choose an integer m > n k kαxk k. Then
α α2 αm  α α2 αm 
1 1
x = x1 + x2 + · · · + xn + ··· + x1 + x2 + · · · + xn ,
m m m m m m
where the right hand side contains m equal groups of summands. By the choice of m, each
partial sum has norm ≤ 2kxk. We thus achieve the desired representation (2.42), with N =
mn.

26. The implication (ii) =⇒ (i) is clear.


To prove the converse implication (i) =⇒ (ii) we proceed as follows. Assume x ∈ span(S),
so that x = limn→∞ yn with yn ∈ span(S) for every n. It is not restrictive to assume
kyn+1 − yn k < 2−n for every n ≥ 1. We can write x as an absolutely convergent sum:

X
x = y1 + (yn+1 − yn ) , yn+1 − yn ∈ span(S).
n=1

Using problem 25, we can write


N (n)
X
yn+1 − yn = cni xni , cni ∈ K , xni ∈ S ,
i=1

with

Xk
cni xni ≤ 2kyn+1 − yn k ≤ 21−n for every k ∈ {1, . . . , N (n)}. (5)



i=1

We now arrange all terms in the double series1

∞ N
X X (n)
cni xni
n=1 i=1

into a single sequence, namely

c11 x11 + · · · + c1,N (1) x1,N (1) + c21 x21 + · · · + c2,N (2) x2,N (2) + c31 x31 + · · ·

Thanks to (5) we obtain a series converging to x.


1
Note that, in general, this double series may not be absolutely convergent. Hence different rearrangements
of its terms may yield different limits. It is the property (5) that guarantees that the sequence converges to x.

13
27. (i) Let V ⊂ `∞ be the subspace of all sequences x = (x1 , x2 , . . .) such that limn→∞ xn
.
exists. For x ∈ V define the linear operator F (x) = limn→∞ xn . Observing that |F (x)| =
| limn→∞ xn | ≤ supn |xn | = kxk∞ ` . it is clear that f : V 7→ R is a continuous linear
functional with norm kF k = 1. Using the Hahn-Banach extension theorem, we can extend F
to the entire space `∞ , still with the same norm kF k = 1.
(ii) Assume m = lim inf xn < lim sup xn = M . If F (x) = M + δ for some δ > 0, a
contradiction is achieved as follows. Choose N large enough so that m − 2δ < xn < M + 2δ for
all n > N . Consider the sequence y = (x1 , x2 , . . . , xN , m, m, m, . . .). Then
δ
F (x) − F (y) = (M + δ) − m > M + − m ≥ kx − yk`∞ .
2
This is a contradiction, because kF k = 1, hence we should have F (x) − F (y)| ≤ kx − yk`∞ .
The case F (x) < m is handled in a similar way.
(iii) Using linearity and then (ii), we obtain
F (y) − F (x) = F (y − x) ≥ lim inf yn − xn ≥ 0 .
n→∞

(iv) If a = (a1 , a2 , . . .) ∈ `1 , then there exists N large enough so that n>N |an | < 1. Consider
P
the sequence x = (0, 0, . . . , 0, 1, 1, 1, . . .), where the first N entries are zero and all other entries
are equal to 1. Then

X X
an xn = an < 1 = lim xn = F (x).
n→∞
n=1 n>N

(v) The functional Fe satisfies (2.43)-(2.44), but it is not linear. For example, take x =
(1, 0, 0, 1, 0, 0, 1, 0, 0, . . .), y = (0, 0, 1, 0, 0, 1, 0, 0, 1, . . .). Then x + y = (1, 0, 1, 1, 0, 1, 1, 0, 1, . . .)
and
1 1 1
F (x + y) = 6= F (x) + F (y) = + .
2 2 2

28. The map f 7→ f (0) is a bounded linear operator on V , with norm


sup |f (0)| = 1 .
f continuous, kf kL∞ ≤1

By the Hahn-Banach theorem it can be extended to a linear functional Λ defined on the entire
space L∞ (R), with norm kΛk = 1.
Next, assume that there exists g ∈ L1 (R) such that Λf = f g dx for all f ∈ L∞ (R). The
R

dominated convergence theorem yields


Z ε
lim |g(x)| dx = 0.
ε→0 −ε

Hence we can choose δ > 0 so that −δ |g(x)| dx ≤ 1/2. Let φ : R 7→ [0, 1] be a smooth
function, with φ(0) = 1 and with support contained in the interval [−δ, δ]. We now reach a
contradiction by writing
Z Z δ Z δ
1
1 = φ(0) = Λ(φ) = g(x)φ(x) dx = g(x)φ(x) dx ≤ max |φ(x)|· |g(x)| dx ≤ .
R −δ x −δ 2

14
29. (i) Assuming that x ∈ λ1 Ω, y ∈ λ2 Ω for some λ1 , λ2 > 0, we claim that x+y ∈ (λ1 +λ2 )Ω.
Indeed, by assumption there exist ω1 , ω2 ∈ Ω such that x = λ1 ω1 , y = λ2 ω2 . Using the
assumption that Ω we obtain
 
λ1 λ2
x + y = λ1 ω1 + λ2 ω2 = (λ1 + λ2 ) · ω1 + ω2 ∈ (λ1 + λ2 )Ω .
λ1 + λ2 λ1 + λ2

The identity p(tx) = tp(x) for t > 0 follows from the fact that x ∈ λΩ if and only if tx ∈ tλΩ.
(ii) We have

kxk
p(x) = inf{λ > 0 ; x ∈ λΩ} ≤ inf{λ > 0 ; x ∈ λBr } = .
r

(iii) If Ω = B1 , then

p(x) = inf{λ > 0 ; x ∈ λB1 } ≤ inf{λ > 0 ; kxk < λ} = kxk .

30. (i) If (fn )n≥1 is a sequence of continuous functions such that fn → f uniformly on [0, 1]
and fn (0) = 0 for every n ≥ 1, then f (0) = 0. Hence X is a closed subspace.
. R1
(ii) The map f 7→ Λf = 0 f (x) dx is a linear functional on X. for every f ∈ X with
R1
kf k = maxx∈[0,1] |f (x)| ≤ 1 we have |Λf | = | 0 f (x) dx| < 1, because f (0) = 0, hence
|f (x)| < 1/2 for x in some interval [0, δ]. On the other hand, choosing fn = x1/n we have
n
fn ∈ X, kfn k = 1 and Λfn = n+1 → 1 as n → ∞. Hence
1
Z
.
kΛk = sup
f (x) dx = 1,
f ∈X , kf k≤1 0

but the supremum is not attained.

31. Being the intersection of a family of closed, convex sets, the set Ω∗∗ is also closed and
convex. It remains to show that Ω∗∗ is contained in the closure of Ω. Assume, on the contrary,
that there exists x0 ∈ Ω∗∗ and a radius r > 0 such that the open ball B(x0 , r) does not
intersect Ω. By the separation theorem, there exists a bounded linear functional F : X 7→ R
and c > 0 such that

F (x) ≤ c for all x ∈ Ω, F (x) > c for all x ∈ B(x0 , r) .

Defining φ(x) = 1c F (x) we obtain

φ(x) ≤ 1 for all x ∈ Ω .

This yields a contradiction, because φ ∈ Ω∗ but φ(x0 ) > 1.

32. On the one dimensional space U = {tξ ; t ∈ R} define the functional F (tξ) = t. This
is a bounded linear functional with norm kF k = kξk. Using the Hahn-Banach Theorem, we
extend F to a bounded linear functional F : X 7→ R with the same norm. Let V = ker(F ).
We claim that all conclusions are satisfied.

15
Since F is continuous linear functional, V is a closed subspace. Given x ∈ X, consider the
projections
. .
πU x = F (x) ξ , πV x = x − F (x)ξ .
These are bounded linear operators, and satisfy

πU x ∈ U , πV x ∈ V , πU x + πV x = x .

33. Consider the subspace Y = {ax + b ; a, b ∈ R} of all polynomials of degree 1. Restricted


to Y , the linear functional ϕ is continuous and has norm

|ϕ(f )| a
kϕk = sup = sup 1 = 1
kf kL1
Z
f ∈Y, f 6=0 (a,b)6=(0,0)
|ax + b| dx
−1

Indeed, for a given a, the minimum of the denominator is achieved when b = 0.


Using the Hahn-Banach theorem, the functional ϕ can then be extended to the entire space
L1 ([−1, 1]), with the same norm.
An explicit form of this functional is
Z 1 Z 0
ϕ(f ) = f (x) dx − f (x) dx .
0 −1

34. It is clear that each pk (·) is a seminorm. If f ∈ C m (Ω), f 6= 0, then f (x) 6= 0 for some
point x ∈ Ω. If x ∈ Ak , then pk (f ) > 0.
To show that the space C m (Ω) is complete for the corresponding distance

. X −k pk (f − g)
d(f, g) = 2 ,
1 + pk (f − g)
k=1

let (fj )j≥1 be a Cauchy sequence. Then for every multi-index α with |α| ≤ m the sequence
of continuous functions (Dα fj )j≥1 is uniformly convergent on each open set Ak . Therefore, it
has a continuous limit, say

f (x) → g(x) , Dα fj (x) → gα (x) for all x ∈ Ω, |α| ≤ m ,

for some continuous functions g, gα . Since the limit is uniform on compact sets, it follows
gα = Dα g for every |α| ≤ m. Hence d(fj , g) → 0, proving that this Cauchy sequence has a
limit.

.
35. Consider the subspace Ye = {ỹ = y + tx0 ; y ∈ Y, t ∈ R} be the space spanned by Y
.
together with x + 0. Let α = d(x0 , Y ) > 0. Define the functional ϕ : Ye 7→ R by setting
.
ϕ(y + tx0 ) = αt .

This clearly implies ϕ(x0 ) = α = d(x0 , Y ) and ϕ(y) = 0 for y ∈ Y .

16
We claim that ϕ is a bounded operator with norm kϕk = 1. Indeed, assume that ky0 + tx0 k <
αt for some choice of y0 ∈ Y and t 6= 0. This leads to a contradiction, because

k − y0 − tx0 k = ky0 + tx0 k < αt ,


y
0
− − x0 < α = inf ky − x0 k .

t y∈Y

Using the Hahn-Banch theorem, the functional ϕ can then be extended to the entire space X.

P (i) If 1 ≤ p < ∞, every continuous linear


36. functional on `p has the form x 7→ ϕ(x) =
q
n an xn , for some sequence a = (an )n≥1 ∈ ` . In this case,

lim ϕ(ek ) = lim ak = 0 .


k→∞ k→∞

proving the weak convergence ek * 0 in the space `p .


(ii) Now consider the case p = ∞. Take any subsequence (enk )k≥1 . Define the element
a = (a1 , a2 , . . .) ∈ `∞ as follows.

1 if j = nk for some odd integer k,
aj =
0 otherwise.
. P
Consider the bounded linear functional on `1 defined by ϕ(x) = j aj xj . Then

lim inf ϕ(enk ) = 0 < 1 = lim sup ϕ(enk ) ,


k→∞ k→∞

showing that the subsequence (enk )k≥1 is not weakly convergent.

37. In the case 1 ≤ p < ∞, assume φ0 (x) ≥ c > 0 for all x ∈ R. For every f ∈ Lp (R), setting
y = φ(x) one obtains
Z 1/p Z 1/p
p p dy 1
|f (φ(x))| dx = |f (y)| 0 ≤ · kf kLp .
φ (x) c1/p

Hence, in this case, Λ is a bounded linear operator on Lp with norm kΛk ≤ c−1/p .
In the case p = ∞, it suffices to assume that φ0 (x) > 0. Then Λ is a bounded
n operator with
∞ . .
norm kΛk = 1. Indeed, if f ∈ L , let M = kf kL∞ . Consider the set A = x ; f (φ(x)) >
o
M . If meas(A) > 0, we obtain a contradiction by writing
  Z
0 = meas φ(A) = φ0 (x) dx > 0.
A

Hence kΛf kL∞ ≤ M . This proves that kΛk ≤ 1. On the other hand, taking the function
g(x) ≡ 1, one has kΛgkL∞ = kgkL∞ = 1.

38. If the Banach space X is infinite dimensional, the closed unit ball is not precompact. In
particular, we can find a sequence of points xn ∈ X such that kxn k ≤ 1, kxm − xn k ≥ 1/2
whenever m 6= n.

17
Choose x̄ ∈ Ω0 and r > 0 such that the ball B(x̄, r) is contained in Ω0 . By assumption, this
implies 0 < µ(B(x̄, r)) ≤ µ(Ω0 ) < ∞.
.
 
Consider the countably many open balls Bn = B 2r xn , 8r . These are mutually disjoint, and
all contained in the open ball B(0, r). Hence, by countable additivity we have
 
[ X
µ(B(0, r)) ≥ µ  Bn  = µ(Bn ) = ∞
n≥1 n≥1

because all balls Bn have the same radius and the same strictly positive measure. This yields
a contradiction, because by translation invariance µ(B(0, r)) = µ(B(x̄, r)) < ∞.

39. (i) This is an immediate consequence of Theorem 2.33 (ii), taking A = {y} and B = S.
/ S. Then there exists a bounded linear functional ϕ ∈ X ∗ such that ϕ(y) <
(ii) Assume y ∈
inf x∈S ϕ(x).
The weak convergence xn * y implies ϕ(xn ) → ϕ(y). But this yields a contradiction because
lim ϕ(xn ) ≥ inf ϕ(x) > ϕ(y).
n→∞ x∈S

40. (i) Let V ⊂ L∞ (R) be the subspace of all functions f such that ess-limx→0 f (x) exists. We
.
claim that the functional Φ(f ) = ess-limx→0 f (x) is a bounded linear functional on V . The
linearity is obvious. If Φ(f ) = λ, then for any ε > 0 there exists δ > 0 such that |f (x) − λ| < ε
for a.e. x ∈ [−δ, δ]. In particular, this implies kf kL∞ ≥ |λ| − ε. Since ε > 0 is arbitrary, we
conclude
|Φ(f )| ≤ kf kL∞ (6)
for all f ∈ V . By the Hahn-Banach theorem, we can extend the linear functional Φ to the
entire space L∞ (R), still satisfying the bound (6) for all f ∈ L∞ (R).
(ii) In the space L1 (R) the conclusion fails because the functional Φ : V 7→ R is unbounded.
Indeed, consider the sequence of continuous functions:

. n(1 − nx) if |x| ≤ 1/n ,
fn (x) =
0 otherwise.
Then fn ∈ V and kfn kL1 = 1 for every n ≥ 1, but Φ(fn ) = fn (0) = n.

Chapter 3

1. Repeat steps 5 - 6 in the proof of the Stone-Weiertrass theorem.

2. (i) True. For every n ≥ 1, let pn be a polymomial such that maxx∈[−n,n] |f (x) −
pn (x)| < 2−n . Such a polynomial exists by the Stone-Weierstrass theorem. The sequence of
polynomials (pn )n≥1 converges to f uniformly on bounded sets.

18
2
(ii) False. If p is a polynomial such that supx∈R |p(x) − e−x | < ∞, then p must be bounded,
2
hence constant. But there is no way to approximate the function f (x) = e−x with constant
functions, uniformly on R.

3. (i) Extend g to an even function defined on [−π, π], and then by periodicity to an even
function defined on the whole real line, periodic of period 2π. By Corollary 3.9 there exists a
trigonometric polynomial of the form
N
X N
X
p(x) = ak sin kx + bk cos kx
k=1 k=0

such that |p(x) − g(x)| < ε for all x ∈ R. Since g is even, this implies
N
p(x) + p(−x) g(x) + g(−x) X
ε > − = b cos kx − g(x) . for all x ∈ R .

k
2 2
k=0

(ii) Extend g to an odd function defined on [−π, π], and then by periodicity to an odd func-
tion defined on the whole real line, periodic of period 2π. By Corollary 3.9 there exists a
trigonometric polynomial of the form
N
X N
X
p(x) = ak sin kx + bk cos kx
k=1 k=1

such that |p(x) − g(x)| < ε for all x ∈ R. Since g is odd, this implies
N
p(x) − p(−x) g(x) − g(−x) X
ε > − = a sin kx − g(x) . for all x ∈ R .

k
2 2
k=1

4. By the Stone-Weierstrass theorem, for any n ≥ 1 there exists a polynomial qn such that
|qn (x) − f 0 (x)| ≤ 2−n for every x ∈ [a, b]. Define the polynomial
Z x
.
pn (x) = f (a) + qn (y) dy.
a

This construction yields

|pn (x) − f (x)| ≤ 2−n (x − a) ≤ 2−n (b − a) , |p0n (x) − f 0 (x)| ≤ 2−n

for every n ≥ 1 and x ∈ [a, b].

5. The properties (i) and (ii) are easily checked. However, (iii) fails. For example, the
function f (θ) = e−iθ cannot be uniformly approximated with functions in the algebra A.
Indeed, assume that
N

X 1
f (θ) − cn einθ ≤ for all θ ∈ [0, 2π].

2
n=0

19
This leads to a contradiction because
Z
N
!
2π X Z 2π
1 iθ
inθ iθ
f (θ) − c e e dθ ≤ |e | dθ ≤ π .

n
2

0 0
n=0

On the other hand, since f (θ) = e−iθ , an explicit computation yields


N N Z 2π
Z 2π ! Z 2π
X X
−iθ inθ iθ
e − cn e e dθ = 1 dθ − cn ei(n+1)θ dθ = 2π .
0 n=0 0 n=0 0

6. Given f ∈ Lp (Ω), extend f to the entire space Rn by setting f (x) = 0 if x ∈


/ Ω.
For any ε > 0, using a mollification we obtain a smooth function f˜ = f ∗ Jδ such that
kf˜ − f kLp (Rn ) < ε.

Since Ω is bounded, its closure Ω is compact. By Corollary 3.6, the continuous function f˜ can
be uniformly approximated by polynomials on the compact set Ω. In particular, there exists
a polynomial p(x) = p(x1 , . . . , xn ) such that

|p(x) − f˜(x)| ≤ ε for all x ∈ Ω .

The previous inequalities imply


Z 1/p
kf − pkLp (Ω) ≤ kf − f˜kLp (Ω) + kf˜ − pkLp (Ω) ≤ ε + p
ε dx = ε + ε(meas(Ω))1/p .

Since ε > 0 was arbitrary, the result is proved.

7. (i) Choose δ > 0 such that |f (x, y) − f (x0 , y)| ≤ ε whenever |x − x0 | ≤ δ. Construct a
continuous partition of unity {g1 , g2 , . . . , gN } on the interval [0, a] such that each gi is supported
on some interval Ii ⊆ [a, b] of length ≤ δ. That means
N
X
gi (x) = 1 for each x ∈ [0, a], gi (x) = 0 for x ∈
/ Ii .
i=1

For each i ∈ {1, . . . , N }, choose a point xi ∈ Ii . For every (x, y) ∈ Q = [0, a] × [0, b] we now
have
XN N
X
f (x, y) − gi (x) f (xi , y) ≤ gi (x) f (x, y) − f (xi , y) ≤ ε. (7)


i=1 i=1
Indeed, the only nonzero terms in the second summation are those for which x ∈ Ii , hence
|x − xi | ≤ δ. By construction, this implies |f (x, y) − f (xi , y)| ≤ ε.
.
Defining hi (y) = f (xi , y), all requirements are satisfied.
(ii) Let 0 < ε ≤ 1 be given. If f vanishes on the boundary of Q, then in the above construction
we can choose xi = 0 if 0 ∈ Ii and xi = a if a ∈ Ii . In both cases, f (xi , y) = 0. Therefore, in
the sum (7) such terms gi (x)f (xi , y) can be discarded, because these vanish identically. As a
result, it is not restrictive to assume that in (7) every gi is a continuous function with support
contained strictly inside the open interval ]0, a[.

20
Since all functions gi vanish at x = 0 and at x = a, and all functions f (xi , y) vanish at y = 0
and at y = b, using problem 3.(ii), we can now find coefficients Aim , Bim and an integer M
such that
M
X πmx
gi (x) − Aim sin ≤ ε for all x ∈ [0, a] ,

a
m=1

M
X πny
f (xi , y) − Bin sin ≤ ε for all y ∈ [0, b] .

b
n=1
This implies

M

gi (x)f (xi , y) −
X πmx πny
Aim Bin sin sin
a b
m,n=1

M
M M

X πmx X πmx X πny
≤ gi (x) − Aim sin |f (xi , y)| + Aim sin · f (xi , y) − Bin sin

a a b


m=1 m=1 n=1

≤ ε · kf kC 0 + (1 + kgi kC 0 ) · ε ≤ (2 + kf kC 0 ) ε .
. P
Therefore, setting cmn = N i=1 Aim Bin , we obtain

N M
X X πmx πny
gi (x)f (xi , y) − cmn sin sin ≤ N (2 + kf kC 0 ) ε . (8)
a b
i=1 m,n=1

Since ε > 0 was arbitrary, by (7) and (8) the result is proved.

8. Let (fn )n≥1 be a sequence of functions in the Hölder space C 0,γ (Ω), with kfn kC 0,γ ≤ C for
some constant C and every n ≥ 1.
Being Hölder continuous, each function fn is uniformly continuous and hence can be uniquely
extended by continuity to the closure Ω, which is a compact set.
Since the functions fn are uniformly bounded and equicontinuous on the compact set Ω, by
Ascoli’s theorem we can extract a subsequence converging to some continuous function f ,
uniformly for x ∈ Ω.

9. (i) f1 ∈ C 0,1/3 . For any 0 < a < b we have


Z b Z b
1 −2/3 1
f1 (b) − f1 (a) = x dx ≤ (x − a)−2/3 dx = (b − a)1/3 .
a 3 a 3
Hence
|f (b) − f (a)|
sup 1/3
≤ 1.
0<a<b<1 |b − a|

On the other hand, choosing an = n−2 , bn = n−1 , for every γ > 1/3 we have
1/3 1/3
bn − an 1
lim = lim = ∞.
n→∞ |bn − an |1/3 · |bn − an |γ−1/3 n→∞ |bn − an |γ−1/3

21
/ C 0,γ for any γ > 1/3.
Therefore f1 ∈

1 1
(ii) f2 ∈ / C 0,γ for any γ > 1. Indeed, choose an = (n+1)π bn = nπ . observing that sin a1n =
− sin b1n = ±1, we compute
p p
|f2 (bn ) − f2 (an )| 1/(n + 1) + 1/n 1
= ≥ .
|bn − an |1/4 (n + n2 )−1/4 2

/ C 0,γ for any γ > 1/4. The fact that f2 ∈ C 0,1/4 is proved by
This already shows that f2 ∈
estimating the ratio
|f2 (b) − f2 (a)|
|b − a|1/4
separately in two cases:
h i h i
1 1 1
CASE 1: b ∈ nπ , (n−1)π , a ∈ (n+1)π , b .
h i
1 1 1
CASE 2: b ∈ nπ , (n−1)π , a< (n+1)π .

The first case is handled simply by integrating |f20 | on the interval [a, b], observing that
|f20 (x)| ≤ x−1/2 + x−3/2 . Case 2 follows from Case 1, using the inequality
 i
h 1
|f2 (b) − f2 (a)| ≤ sup |f2 (b) − f2 (x)| ; x ∈ , b .
(n + 1)π

(iii) f3 ∈ C 0,γ for every γ ∈ ]0, 1[ . Indeed, for any 0 < a < b < 1 one has
Z b Z b Z b
γ−1 Cγ
f3 (b) − f3 (a) = (| ln x| − 1) dx ≤ Cγ x dx ≤ Cγ (x − a)γ−1 dx ≤ (b − a)γ ,
a a a γ

for some constant Cγ .

10. Ascoli’s theorem guarantees the existence of a subsequence (fnk )k≥1 which converges to
some limit function f ∈ C 0,γ ([0, 1]), uniformly on [0, 1]. This means kfnk −f kC 0 → 0. However,
in general it is not true that kfnk − f kC 0,γ → 0. In other words, the convergence takes place
only in the norm of C 0 , not in the stronger norm of C 0,γ .

11. To show that k · kϕ is a norm, we need to check that it satisfies the conditions (N1)–(N3)
in the definition. The first two conditions are clear. To prove (N3) we observe that, for any
x, y ∈ Ω, x 6= y, one has
|(f + g)(x)| ≤ |f (x)| + |g(x)| ,
|f (x) + g(x) − f (y) − g(y)| |f (x) − f (y)| |g(x) − g(y)|
≤ + .
ϕ(|x − y|) ϕ(|x − y|) ϕ(|x − y|)
Taking the supremum over all x, y we obtain kf + gkϕ ≤ kf kϕ + kgkϕ .
It remains to show that the space C ϕ is complete. For this purpose, let (fn )n≥1 be a Cauchy
sequence w.r.t. the norm k · kϕ . Then this sequence is Cauchy w.r.t. the C 0 norm, hence it

22
converges uniformly to a continuous function f : Ω 7→ R. To show that kfn −f kϕ → 0, observe
that, for any ε > 0 there exists N large enough so that

fn (x) − fm (x) − fn (y) + fm (y)

sup < ε for all m, n ≥ N .
x6=y |x − y|

Keeping n fixed and letting m → ∞, this implies



f (x) − f (x) − f (y) + f (y)

n n
sup < ε for all n ≥ N .
x6=y |x − y|

Hence kfn − f kϕ → 0.

12. One simply needs to retrace all steps in the proof of Ascoli’s theorem. Since E is compact,
given ε > 0 there exists δ > 0 such that

d(x, y) ≤ δ =⇒ |f (x) − f (y)| ≤ ε for all x, y ∈ E and f ∈ F.

SinceSK is compact, it is precompact. Hence we can choose points α1 , . . . , αm ∈ K such that


K⊆ m j=1 B(αj , ε). The remaining steps of the proof can be repeated without change.

Chapter 4

1. (1) The operator Λ is linear and bounded, with kΛk = 1, but not compact.
(2) This operator is not linear: (Λ(2f ))(x) = sin(2f (x)) 6= 2 sin(f (x)) = 2(Λf )(x).
(3) Λ is linear and bounded, with kΛk = 1, but not compact.
(4) Λ is linear, bounded, and compact. Indeed, Λf is always a polynomial of degree ≤ 1.
Hence the range of Λ is two-dimensional.
(5) This operator Λ is linear, bounded, and compact. Indeed,
Z x
(Λf )(x) = ey−x f (y) dy ,
0

Hence, for any x ∈ [0, 1],


Z x
|(Λf )(x)| ≤ |f (y)| dy ≤ kf |C .
0

To see that Λ is compact, let (fn )n≥1 be a sequence of continuous functions with kfn kC ≤ 1
for every n. Then all functions Λ(fn ) are Lipschitz continuous with constant 2. Indeed, from
the differential equation it follows

|y 0 (x)| ≤ |f (x)| + |y(x)| ≤ 1 + 1 .

23
By Ascoli’s theorem, the sequence (fn ) admits a uniformly convergent subsequence.

2. To show that En is closed, assume kfk − f kL1 → 0 and kfk k2L2 ≤ n for every k. By possibly
taking a subsequence we can assume fk (x) → f (x) for a.e. x ∈ [0, 1]. Fatou’s lemma now
yields Z 1 Z 1 Z 1
|f (x)|2 dx = lim inf |fk (x)|2 dx ≤ lim inf |fk (x)|2 dx ≤ n .
0 0 n→∞ n→∞ 0
Hence f ∈ En as well.
To prove that En has empty interior in L1 ([0, 1]), consider any f ∈ En . For any ε > 0, define

f (x) + εx−1/2

. if f (x) ≥ 0 ,
fε (x) = −1/2
f (x) − εx if f (x) < 0 .

Then kfε − f kL1 = 2ε but |fε (x)| ≥ εx−1/2 for every x ∈ ]0, 1], and hence fε ∈
/ L2 ([0, 1]).

3. Let πn : `∞ 7→ R be the projection operator, so that πn (x1 , x2 , . . .) = xn . If Λ : X 7→ `∞ is


bounded, then the composition Λn = πn ◦ Λ is bounded as well.
Viceversa, assume that each linear functional Λn is bounded. By assumption, for every x ∈ X
one has
sup |Λn (x)| < ∞ .
n≥1

The uniform boundedness principle thus implies kΛk = supn≥1 kΛn k < ∞.

4. By the closed graph theorem, it suffices to prove that Λ has closed graph. Toward this
goal, consider a sequence of functions fn ∈ Lp ([0, 1]) such that

kfn − f kLp → 0 , kΛ(fn ) − gkLp → 0 ,

for some functions f, g ∈ Lp ([0, 1]). We need to show that Λf = g. By choosing a subsequence
and relabeling, we can assume the pointwise convergence

fn (x) → f (x) , (Λfn )(x) → g(x) for a.e. x ∈ [0, 1] .

By assumption, this implies (Λfn )(x) → (Λf )(x) for a.e. x. Hence (Λf )(x) = g(x) for a.e.
x ∈ [0, 1].

5. Assume that the range Y = K(X) is a closed subspace of X. Then Y itself is complete.
The linear operator K is a continuous bijection between X and the Banach space Y . Hence
it has a continuous inverse K −1 .
Being the composition of a compact operator and a continuous one, the identity map ι(x) =
x = K −1 K(x) is a compact operator from X into itself. But this is impossible, because X is
infinite dimensional.

6. Let (xn )n≥1 be a bounded sequence of points in X.


CASE 1: Λ1 compact. Then the sequence Λ1 (xn ) admits a convergent subsequence, say
Λ1 (xnk ) → y. Since Λ2 is continuous, this implies the convergence Λ2 ◦ Λ1 (xnk ) → Λ2 (y).

24
CASE 2: Λ2 is compact. Since Λ1 is continuous, the sequence (Λ1 (xn ))n≥1 is bounded in Y .
By the compactness of Λ2 , the sequence (Λ2 ◦ Λ1 (xn ))n≥1 admits a convergent subsequence.

.
7. If Y = Range(Λ) is an infinite dimensional normed space, then the unit ball in Y is not
precompact. As shown in the proof of Theorem 2.22, there exist a sequence of points yn ∈ Y
such that kyn k ≤ 1, kyn − ym k ≥ 1/2 for all m, n ≥ 1 m 6= n. Hence from this sequence
(yn )n≥1 one cannot extract any convergent subsequence.
This leads a contradiction, because the assumptions imply that Λ is compact and Λ(y) = y
for every y ∈ Y .

.
8. First of all, observe that the assumptions imply 0 ∈ U . Call V = −U ∩ U . Since V ⊆ U ,
it suffices to prove that V contains a neighborhood of the origin. Observe that
(i) V is closed, convex.
(ii) V = −V , 0 ∈ V .
S
(iii) n≥1 nV = X.
Indeed, (i)-(ii) are clear. To prove (iii), consider any x ∈ X. Then there exist integers n1 , n2
.
large enough so that x ∈ n1 U , −x ∈ n2 U . Calling n = max{n1 , n2 } we have nx ∈ U , − nx ∈ U .
By convexity, this implies
nx xo 1o
.
n
A = co , − = θx ; |θ| ≤ ⊆ U.
n n n
Since A = −A, we conclude that A ⊆ V . Hence x ∈ nA ⊆ nV .
S
Since X is a Banach space, X = n≥1 nV , and each set nV is closed, by Baire’s category
theorem at least one of the sets nV must have non-empty interior. By homogeneity, V itself
has non-empty interior. If B(x, r) ⊆ V , then (ii) implies B(−x, r) ⊆ V . By convexity,
1 1
B(0, r) = B(x, r) + B(−x, r) ⊆ V.
2 2

9. Assume that K is surjective. By the open mapping theorem, K is an open map. In


.
particular, the image of the unit ball B1 = {x ∈ X ; kxk < 1} contains a neighborhood of
the origin. But this is impossible, because the closure K(B1 ) is compact, and cannot contain
any open set in the infinite dimensional space Y .
 
In the particular example, the point y = 1, 41 , 91 , . . . , n12 , . . . lies in `1 but not in the image
K(`1 ).

10. Fix any x ∈ R and consider a sequence xn → x. Since f is bounded, there exists the
upper and lower limits
. .
m = lim inf f (y) ≤ lim sup f (y) = M .
y→x y→x

We claim that m = M = f (x). Indeed, assume m 6= f (x). Then we can find a sequence
xn → x such that f (xn ) → m. Hence the point (x, m) lies in the closure of the graph of f . If

25
f (x) 6= m, this graph is not closed and we reach a contradiction. The proof that f (x) = M is
entirely similar.
To show that the result fails without the boundedness assumption, consider the function
f (x) = x−1 if x 6= 0, f (0) = 0. Then f has closed graph but is not continuous at x = 0.

11. Let f : X 7→ Y be continuous. Assume that xn → x and f (xn ) → y. By continuity,


f (x) = limn→∞ f (xn ) = y, hence the graph of f is closed.
To construct the example, choose any function 0 6= f ∈ L1 (R). Define the map g : t 7→ g t (·)
from R into L1 (R) by setting

 0 if t ≤ 0, x ∈ R ,
.

t
g (x) =
 f x− 1
 
if t > 0, x ∈ R .

t
Then g is not continuous at t = 0 because, for t > 0, kg t − g 0 kL1 = kf kL1 and does not
approach zero as t → 0+. On the other hand, the map g has closed graph. To see this, simply
observe that, for any sequence tk → 0+, the sequence f tk has no limit.

12. Assume that limk→∞ λk → 0. For each n ≥ 1 consider the truncated operator
Λn (x1 , x2 , . . .) = (λ1 x1 , λ2 x2 , . . . , λn xn , 0, 0, . . .). (9)
Each Λn is continuous and has finite dimensional range, hence is compact. Observing that
kΛ − Λn k = supj>n |λj | → 0 as n → ∞, by Theorem 4.10 we conclude that Λ is compact.
On the other hand, assume that lim supn→∞ |λn | > 0. Then there exists ε > 0 and a subse-
quence (λnk )k≥1 such that |λnk | ≥ ε for every k ≥ 1. Consider the unit vectors
e1 = (1, 0, 0, 0, . . .), e2 = (0, 1, 0, 0, . . .), e3 = (0, 0, 1, 0, . . .), ...
Then the sequence (enk )k≥1 is bounded, but the sequence Λ(enk ) = λnk enk does not admit
any convergent subsequence. Indeed,
kΛenj − Λenk k = kλnj enj − λnk enk k ≥ kλnk enk k ≥ ε .
Hence Λ is not compact.

13. Let B be continuous at the origin. Then there exists δ > 0 such that |B(x, y)| ≤ 1
whenever kxk ≤ δ and kyk ≤ δ.
For any (x, y) ∈ X × Y we now have
 
1 δx δy
|B(x, y)| = 2 · kxk kyk · B , ≤ C · kxk kyk ,
δ kxk kyk
with C = δ −2 .

14. Given any polynomial p, we have


Z 1 Z 1

p(t) q(t) dt ≤ max |p(x)| · |q(t)| dt.

0
x∈[0,1] 0

26
Hence the map Λp : q 7→ B(p, q) is a bounded linear operator from X into itself, with norm
kΛp k = kpkC([0,1]) .
To prove that the bilinear map B is not continuous on the product space X ×X, we proceed as
follows. Choose a sequence of numbers xn > 0 such that | ln xn | > n2 . Define the continuous
functions  1
 √ if x ∈ [xn , 1] ,
nx


.

fn (x) =
 1
if x ∈ [0, xn ] .

 √

nxn
Observe that Z 1 Z 1

1 2
fn (x) dx < √ x dx = √ .
0 n 0 n
Z 1 Z 1
1 1
fn2 (x) dx > dx > | ln xn | ≥ n .
0 xn nx n
By the Stone-Weierstrass theorem, each function fn can be uniformly approximated by a
polynomial. We can thus construct a sequence of polynomials (pn )n≥1 such that
Z 1 Z 1
2
pn (x) dx < √ , p2n (x) dx > n
0 n 0

for every n ≥ 1. We thus have (pn , pn ) → 0 in X × X, but B(pn , pn ) → ∞.

15. Since S is bounded, there exists M such that kxk ≤ M for all x ∈ S. Let ε ∈ ]0, 1] be
given. Let Yε be a finite dimensional space such that d(x, Yε ) ≤ ε for all x ∈ S.
.
In the finite dimensional space Yε , the closed ball B = {y ∈ Yε ; kyk ≤ 1 + M } centered at
the origin with radius 1 + M is compact. Hence it can be covered with finitely many balls of
radius ε, say B(yn , ε), n = 1, . . . , N . We claim that
N
[
S ⊆ B(yn , 2ε) .
n=1

Indeed, for every x ∈ S there exists yx ∈ Yε such that kx − yx k ≤ ε. Clearly, we must have
kyx k ≤ M + ε. Hence yx ∈ B(yn , ε) for some n. This implies x ∈ B(yn , 2ε).
The previous analysis shows that S is precompact. Namely, for every ε > 0 is can be covered
by finitely many balls of radius ε. By assumption, S is a closed subset of a Banach space,
hence it is complete. We thus conclude that S is compact.

16. If A 6= λI, consider two cases.


CASE 1: There exists a vector f ∈ X such that g = Af is not a scalar multiple of f .
Clearly, the two vectors f, g must be linearly independent.
Consider the two-dimensional subspace V = span{f, g}. Let ϕ : V 7→ R be the linear func-
tional such that ϕ(f ) = 0, ϕ(g) = 1. Otherwise stated,
.
ϕ(αf + βg) = β for all α, β ∈ K .

27
Using the Hahn-Banach extension theorem, this functional can be extended to a bounded
linear functional, still called ϕ, defined on the entire space X.
.
We then define the operator K : X 7→ X by setting K(x) = ϕ(x) g. Notice that this implies

K(f ) = ϕ(f ) g = 0, K(g) = ϕ(g) g = g .

Moreover, K has one-dimensional range, hence it is compact.


We now check that A(Kf ) = 0 while K(Af ) = Kg = g. Hence AK 6= KA.

CASE 2: There exists two nonzero vectors f, g ∈ X such that Af = λf , Ag = λ0 g with λ 6= λ0 .


Then A(f + g) = λf + λ0 g is not a scalar multiple of f + g, and Case 1 again applies.

17. Since V is an infinite dimensional normed space, we can find a sequence of points (xn )n≥1
in V such that kxn k ≤ 1, kxm − xn k ≥ 1/2 for all m 6= n. Then the sequence (Λxn )n≥1 does
not admit any convergent subsequence. Indeed, by assumption
ε
kΛxm − Λxn k ≥ εkxm − xn k ≥ for all m 6= n.
2
Therefore, the operator Λ cannot be compact.

18. Consider the family {ϕn ; n ≥ 1} of bounded linear functionals on X ∗ , defined by


n
X

ϕn (x ) = hx∗ , xk i.
k=1

By assumption,
lim ϕn (x∗ ) = ϕ(x∗ )
n→∞

exists for every x∗ ∈ X ∗ . Hence supn≥1 |ϕn (x∗ )| < ∞ for every x∗ ∈ X ∗ .
.
By the uniform boundedness principle, M = supn≥1 kϕn k < ∞. Therefore
.
kϕk = sup |ϕ(x∗ )| ≤ sup sup |ϕn (x∗ )| ≤ sup kϕn k = M,
kx∗ k≤1 kx∗ k≤1 n≥1 n≥1

showing that ϕ is bounded.

19. For every continuous function f : [0, 1] 7→ R, the mean value theorem implies

1 t
Z
lim f (s) ds = f (0) .
t→0 t 0

Hence the function Λf is continuous as well.


(i) It is clear that the operator Λ is linear. Observing that the average value of f over the
interval [0, t] satisfies
Z t
1
f (s) ds ≤ max |f (s)| ≤ kf kC([0,1] ,
t s∈[0,t]
0

28
we conclude that Λ is a bounded operator, with norm kΛk ≤ 1.
Rt
(ii) If (Λf )(x) = 0 for every x ∈ [0, 1], then 0 f (s) ds = 0 for every t > 0 and hence f (s) = 0
for every x ∈ [0, 1]. This proves that Λ is one-to-one.
If g = Λf for some continuous function f , taking derivatives we obtain
f (t) − g(t)
g 0 (t) = for all t ∈ ]0, 1[ .
t
Hence g is continuously differentiable. Any continuous function g : [0, 1] 7→ R which is not
continuously differentiable cannot be in the range of Λ.
(iii) To prove that Λ is not compact, consider the sequence of functions

sin 2n x if x ∈ [0, 21−n π] ,



.
fn (x) =
0 otherwise.

Observe that Λfn (x) = 0 for x ≥ 21−n π. Hence, for m < n a direct computation yields
2 2
kΛfm − Λfn kC 0 ≥ Λfm (2−m π) − Λfn (2−m π) = − 0 = .

π π
As a consequence, the sequence (Λfn )n≥1 does not admit any uniformly convergent subse-
quence.

20. (i) The multiplication operator Mf is one-to-one if and only if g(x) 6= 0 for a.e. x ∈ Ω.
(ii) In general, the range of Mg is not closed. For example, let Ω = ]0, 1[ and g(x) = x. Then
the function h(x) ≡ 1 is not in the range of Mg , because the function f (x) = h(x)/g(x) = 1/x
.
does not lie in Lp (Ω). However, if 1 ≤ p < ∞, then the characteristic functions fn = χ
[1/n , 1]
lie in the range of Mg . Moreover, kfn − f kLp → 0 whenever 1 ≤ p < ∞.
To see a positive result, assume that there exists ε > 0 such that the domain Ω can be
decomposed as Ω = Ω0 ∪ Ωε , with Ω0 , Ωε disjoint measurable sets, and

g(x) = 0 for a.e. x ∈ Ω0 , |g(x)| ≥ ε for a.e. x ∈ Ωε .

Then Λ has closed range. Indeed,


n o
Range(Λ) = f ∈ Lp (Ω) ; f (x) = 0 for a.e. x ∈ Ω0 .

(iii) If g does not coincide a.e. with the zero function, then the operator Mg is not compact.
.
To see this, assume that, for some ε > 0, the set Aε = {x ∈ Ω ; |g(x)| ≥ ε} has strictly
positive measure. Then we can find countably many disjoint measurable sets An such that

An ⊂ Aε , 0 < meas(An ) < 1 for all n ≥ 1 .

Define the sequence of functions



. cn if x ∈ An ,
fn (x) =
0 otherwise,

29
choosing the constants cn so that kfn kLp = 1. By construction, the sequence (fn )n≥1 is
bounded. However, the sequence (Mg fn )n≥1 does not admit any convergent subsequence.
Indeed, for any m 6= n, since fn and fm have disjoint support, one has

kfn g − fm gkLp ≥ kfn gkLp ≥ ε kfn kLp = ε.

21. In the space `1 , consider the compact operator


x x x 
1 2 3
Λ(x1 , x2 , x3 , . . .) = , , , ... .
1 2 3
Then Λ is one-to-one and its range is dense in `1 . However, given any ε > 0, choose n > ε−1
and consider the truncated operator
x x x xn 
1 2 3
Λn (x1 , x2 , x3 , . . .) = , , , ..., , 0, 0, . . . .
1 2 3 n
Then kΛn − Λk < ε but Λn is not one-to-one and its range is not dense in `1 .

22. (i) By the open mapping theorem, the functional Λ has a continuous inverse. Hence there
exists a constant C > 0 such that kΛ−1 yk ≤ C kyk for every y ∈ Y . Taking β = C −1 , this
implies βkxk ≤ kΛxk for all x ∈ X.
.
(ii) We check that, if γ = kΨk < 1/β, then the map u 7→ Λ−1 (f − Ψu) is a strict contraction.
Indeed, for any u, v ∈ X one has

kΛ−1 (f −Ψu)−Λ−1 (f −Ψv)k ≤ kΛ−1 (Ψv)−Λ−1 (Ψu)k ≤ kΛ−1 k·kΨk ku−vk ≤ β · γku−vk .

Since β · γ < 1, this is a strict contraction and therefore has a unique fixed point.
(iii) Let Λ : X 7→ Y be a continuous bijection, and let β > 0 be such that kΛxk ≥ βkxk for all
x ∈ X. We claim that, if Ψ : X 7→ Y is any bounded linear operator with kΨk < β −1 , then
the sum Λ + Ψ is still a bijection. Indeed, by (ii) for any f ∈ Y there exists a unique element
u ∈ X such that u = Λ−1 (f − Ψu). This implies Λu = f − Ψu, hence (Λ + Ψ)u = f . showing
that the operator ΛΨ is a bijection.

23. (i) From the definition it follows



1 Xn
|yn | = xi ≤ max |xi | ≤ kxk`∞ .

n 1≤i≤n
i=1

Hence this averaging operator has norm kΛk = 1.


To prove that Λ is not compact, consider the sequences xk = (xk1 , xk2 , xk3 , . . .) ∈ `∞ defined
by
1 if 2k−1 < i ≤ 2k ,

.
xki =
0 otherwise.
Clearly kxk k`∞ = 1 for every k. However, this bounded sequence does not admit any subse-
quence (xk` )`≥1 such that Λxk` converges. Indeed, if j < k, then
1 1
kΛxj − Λxk k`∞ ≥ |xj,2j − xk,2j | = − 0 = .

2 2

30
(ii) Λis not a bounded operator from `1 into `1 . For example, if x = (1, 0, 0, . . .) ∈ `1 , then
Λx = 1, 21 , 13 , . . . ∈/ `1 .

24. (i) If Y = X, let ι : Y 7→ X be the identity map. This is a bijective, continuous, linear
map from Y onto X. By Corollary 4.5, the inverse map ι−1 : X 7→ Y is continuous as well.
Hence there exists a constant C 0 such that kxkY ≤ C 0 kxkX for all x ∈ X.
.
(ii) Assume Y 6= X. Consider the sets Sn = {y ∈ Y ; kykY ≤ n}, and let S n be the closure
of Sn in the space X. We claim that

(C) Each S n has empty interior.

If (C) holds, then [ [


Y = Sn ⊆ Sn .
n≥1 n≥1

Hence Y is of first category, being contained in the union of countably many closed sets with
empty interior.
To prove (C), we argue by contradiction. If some S n has nonempty interior, by positive
homogeneity the set S 1 has nonempty interior as well. To fix the ideas, assume that the open
ball B(x, r) ⊂ S 1 for some x ∈ S 1 and r > 0. Since S 1 = −S 1 , we also have B(−x, r) ⊂ S 1 .
By convexity, this implies
1 1
B(0, r) = B(x, r) + B(−x, r) ⊂ S 1 .
2 2
In turn, by positive homogeneity this yields
.
B(0, 2−n r) ⊂ S 2−n = {y ∈ Y ; kykY ≤ 2−n } .

Repeating the argument in step 3. of the proof of Theorem 4.4 (the Open Mapping Theorem),
we conclude that B(0, r/2) ⊂ S1 .
For any x ∈ X we thus have

3 kxkX rx 3 kxkX 3 kxkX


x = · ∈ · B(0, r/2) ⊂ · S1 ⊂ Y.
r 3kxkX r r

This shows that X = Y , against the assumption.

Chapter 5

1. Using the properties of inner products one obtains

(x, y + z) = (y + z, x) = (y, x) + (z, x) = (x, y) + (x, z).

31
(x, αy) = (αy, x) = α(y, x) = ᾱ(x, y) .
Saying that the vectors x, y are mutually orthogonal means that (x, y) = 0. This implies

kx + yk2 = (x + y, x + y) = (x, x̄) + (x, ȳ) + (y, x̄) + (y, ȳ) = kxk2 + 0 + 0 + kyk2 .

.
2. Let x, y ∈ H and ε > 0 be given. Let M = max{kxk, kyk}. Choose 0 < δ < 1, such that
δ = ε/(1 + 2M ). If kx0 − xk ≤ δ and ky 0 − yk ≤ δ, then

|(x0 , y 0 ) − (x, y)| = (x0 − x, y 0 − y) + (x0 − x, y) + (x, y 0 − y)

≤ kx0 − xk ky 0 − yk + kx0 − xk kyk + kxk ky 0 − yk ≤ δ 2 + 2M δ ≤ ε .

3. (i) The parallelogram identity is proved by writing

kx+yk2 +kx−yk2 = (x+y, x+y)+(x−y, x − y) = 2(x, x)+2(y, y) = 2kxk2 +2kyk2 . (10)

(ii) Assume that the norm k · k satisfies the parallelogram identity. We claim that the assign-
ment
. 1 1
  
(x, y) = kx + yk2 − kxk2 − kyk2 = kx + yk2 − kx − yk2 (11)
2 4
satisfies all properties of an inner product. Note that the equality between the last two terms
in (11) is an easy consequence of (10).
By (11), the identity (x, y) = (y, x) is obvious.
To check that (x + x0 , y) = (x, y) + (x0 , y), using the parallelogram identity we obtain

kx + x0 + yk2 = 2kx + yk2 + 2kx0 k2 − kx + y − x0 k2

= 2kx0 + yk2 + 2kxk2 − kx0 + y − xk2

1 1
= kxk2 + kx0 k2 + kx + yk2 + kx0 + yk2 − kx + y − x0 k2 − kx0 + y − xk2 .
2 2
This implies
1 
(x + x0 , y) = kx + x0 + yk2 − kx + x0 − yk2
4
 
1 2 0 2 2 0 2 1 0 2 1 0 2
= kxk + kx k + kx + yk + kx + yk − kx + y − x k − kx + y − xk
4 2 2
 
1 2 0 2 2 0 2 1 0 2 1 0 2
− kxk + kx k + kx − yk + kx − yk − kx − y − x k − kx − y − xk
4 2 2

1  1 
= kx + yk2 − kx − yk2 + kx0 + yk2 − kx0 − yk2 = (x, y) + (x0 , y) .
4 4

32
In particular, from the above identity by induction it follows

(nx, y) = (x + x + · · · + x, y) = n(x, y) .
 
1
In turn, this implies n x, y = n1 (x, y), and hence

(λx, y) = λ(x, y) (12)


m
for every rational number λ = n. By continuity, we conclude that (12) remains valid for every
λ ∈ R.
Finally, we check that if x 6= 0, then
1
kx + xk2 − kx − xk2 = kxk2 > 0.

(x, x) =
4
p
Hence the inner product is positive definite, and (x, x) = kxk.

4. (i) Choose x = (1, 0), y = (0, 1). Then kxkp = kykp = 1, kx + ykp = kx − ykp = 21/p . If
p 6= 2, then

kx + yk2 + kx − yk2 = 22/p + 22/p 6= 2 + 2 = 2kxk2 + 2kyk2

(ii) Choose f (x) = sin πx, g(x) = 1 − sin πx. Then kf k = kgk = kf + gk = kf − gk = 1 and
the parallelogram identity fails.

5. Since the monomials 1, x are mutually orthogonal in L1 ([−1, 1]), the first two polynomials
in an orthonormal basis are
r
1 1 x 3
p0 (x) = = √ , p1 (x) = = x.
k1kL2 2 kxkL2 2

To find p2 , we compute the inner product


1 1

x2
Z Z
2 2 2
(p0 , x )L2 = p0 (x) x dx = √ dx = .
−1 −1 2 3

Hence q(x) = x2 − (p0 , x2 )L2 · p0 (x) = x2 − 13 is perpendicular to p0 , and to p1 as well. We


compute
Z 1 Z 1
2 2 1 2 2 1 1 2 1 8
kqkL2 = x − dx = 2 x4 − x2 + dx = 2 − + = .
−1 3 0 3 9 5 9 9 45

Therefore r
q(x) 45  2 1 
p2 (x) = = x − .
kqkL2 8 3

P∞ 2
6. For every x ∈ H, Bessel’s inequality (5.10) implies n=1 |(x, en )| ≤ kxk2 . Hence
limn→∞ |(x, en )| = 0. By definition, this means that en * 0.

33
7. Since H is infinite dimensional, for a given vector x ∈ Hpwe can find an orthonormal
.
sequence (ek )k≥1 such that (x, ek ) = 0 for every k. Choose λ = 1 − kxk2 . Then the sequence
.
xn = x+λen satisfies all requirements. Indeed, by Pythagora’s theorem kxn k2 = kxk2 +λ2 = 1.
Moreover, for any y ∈ H by the previous problem 6. we have

lim (y, xn ) = (y, x) + lim λ(y, en ) = (y, x) .


n→∞ n→∞

P
8. (i) By assumption, k kαk vk k < ∞, hence the series is absolutely convergent. Since H is
complete, the series converges.
(ii) Let (vk )k≥1 be an orthonormal sequence. Then (assuming m ≤ n)

X X
|αk |2 .

lim sup
αk vk = lim sup
m,n→∞ m,n→∞
m<k≤n m<k≤n

|αk |2 < ∞.
P
This shows that the sequence of partial sums is Cauchy if and only if k

9. For any f ∈ L2 (Rn ), performing the change of variable y = φ(x) and using the fact that
detDφ ≡ 1, we compute
Z Z Z
2 2
|(Λf )(x)| dx = |f (φ(x))| dx = |f (y)|2 dy.
Rn Rn Rn

Hence kΛf kL2 = kf kL2 for every f ∈ L2 (Rn ), proving that kΛk = 1.
The adjoint operator Λ∗ is defined by the identity
Z Z Z Z

f (y)·(Λ g)(y) dy = (Λf )(x)·g(x) dx = f (φ(x))·g(x) dx = f (y)·g(φ−1 (y)) dy .
Rn Rn Rn Rn

Hence (Λ∗ g)(y) = g(φ−1 (y), showing that Λ∗ = Λ−1 .

10. The adjoint operator is defined by the identity



X ∞
X
(Λx, y) = xi+1 yi = xj yj−1 = (x, Λ∗ y).
k=1 j=2

Hence Λ∗ (y1 , y2 , y3 , . . .) = (0, y1 , y2 , . . .). Clearly, Λ is onto but not one-to-one, while Λ∗ is
one-to-one but not onto.

11. (i) Assume x = θx1 + (1 − θ)x2 , for some x1 6= x2 , with kx1 k ≤ 1, kx2 k ≤ 1 and 0 < θ < 1.
By the parallelogram’ identity

kx1 + x2 k2 = kx1 k2 + kx2 k2 + 2Re(x1 , x2 ) = 2kx1 k2 + 2kx2 k2 − kx1 − x2 k2 .

If x1 6= x2 , then we have the strict inequality

2Re(x1 , x2 ) < kx1 k2 + kx2 k2 .

34
Therefore  
kxk2 = θx1 + (1 − θ)x2 , θx1 + (1 − θ)x2

= θ2 kx1 k2 + (1 − θ)2 kx2 k2 + 2θ(1 − θ)Re(x1 , x2 )

< θ2 kx1 k2 + (1 − θ)2 kx2 k2 + θ(1 − θ)(kx1 k2 + kx2 k2 )

≤ θ2 + (1 − θ)2 + 2θ(1 − θ) = 1.
This proves that every point x in the closed unit ball of a Hilbert space which is not extremal
must have norm kxk < 1. Hence all points with kxk = 1 are extremal.
(ii) Next, assume f ∈ L1 ([0, 1]) with kf kL1 = 1. Then we can find a set A ⊂ [0, 1] with
positive measure such that |f (x)| ≥ 1/2 for every x ∈ A. To fix the ideas, assume f (x) ≥ 1/2
for all x ∈ A, with meas(A) > 0. Choose two disjoint measurable subsets A1 , A2 ⊂ [0, 1] with
A1 ∪ A2 = A, meas(A1 ) = meas(A2 ). Consider the function

 1/2 if x ∈ A1 ,
g(x) = −1/2 if x ∈ A2 ,
0 if x ∈
/ A1 ∪ A2 .

Then
f +g f −g
kf + gk1L = kf − gkL1 = 1 , f =
+ .
2 2
Hence f is not an extreme point of the unit ball in L1 , being a convex combination of the two
functions f + g and f − g.

12. By taking a subsequence and relabeling, one can assume kxn k → C as n → ∞. Since this
sequence is bounded, by Theorem 5.14 it admits a weakly convergent sequence, say xnj * x
for some x ∈ H. We now compute
kxk2 = (x, x) = lim (x, xnj ) ≤ lim sup kxk kxnj k ≤ Ckxk .
j→∞ j→∞

Dividing by kxk we conclude kxk ≤ C.

13. By assumption, H admits a countable, everywhere dense set {x1 , x2 , x3 , . . .}. Using
the Gram-Schmidt procedure, from this set we construct a countable orthonormal basis
{e1 , e2 , e3 , . . .}.
Defining
. .
Λx = a = (a1 , a2 , a3 , . . .) with ak = (x, ek ),
we obtain the desired bijection. Indeed,

X ∞
X
kΛxk`2 = |ak |2 = |(x, ek )|2 = kxk .
k=1 k=1

.
14. (i) Assume x ∈ V = span{v1 , . . . , vn }, so that
n
X
x = θk v k
k=1

35
v2 v3

h3
h
2
v2
v1 v1

Figure 1: Computing the area of a parallelogram and the volume of a parallelepiped. Here h2 =
d(v2 , span{v1 }) while h3 = d(v3 , span{v1 , v2 }).

for some coefficients θk . To actually compute these coefficients, we observe that, for every
j = 1, . . . , n, one must have
n
X
(x, vj ) = θk (vk , vj ) .
k=1

Therefore the numbers θ1 , . . . , θn are obtained by solving the system of n linear equations
    
(v1 , v1 ) · · · (vn , v1 ) θ1 (x, v1 )
 .. .. ..   ..  =  ..  .
 . . .  .   .  (13)
(v1 , vn ) · · · (vn , vn ) θn (x, vn )

Take x to be the zero vector in (13). Then that the vectors v1 , . . . , vn are linearly independent
if and only if θ1 = θ2 = · · · = θn = 0 is the only solution of (13) when x = 0. This is the case
if and only if the determinant G((v1 , . . . , vn ) of the n × n symmetric matrix in (13) is not zero.
(ii) If x ∈ H and y ∈ V then G(x, v1 , v2 , . . . , vn ) = G(x + y, v1 , v2 , . . . , vn ). In particular,
taking y = PV (x) and z = x − y = PV ⊥ (x), we obtain

d(x, V ) = kzk , G(x, v1 , v2 , . . . , vn ) = G(z, v1 , v2 , . . . , vn ) = (z, z) G(v1 , v2 , . . . , vn ).

Therefore s
1/2 G(x, v1 , v2 , . . . , vn )
d(x, V ) = kzk = (z, z) = .
G(v1 , v2 , . . . , vn )

(iii) The volume of the parallelepiped with edges v1 , . . . , vn can be computed as a product:
     
kvn k · d vn−1 ; span{vn } · d vn−2 ; span{vn−1 , vn } · · · d v1 ; span{v2 , . . . , vn }
s s
p G(vn−1 , vn ) G(v1 , v2 , . . . , vn ) p
G(vn ) · ··· = G(v1 , v2 , . . . , vn ),
G(vn ) G(v2 , . . . , vn )

where each factor was computed using (ii).

15. Let U ⊂ L2 (R) be the subspace of all even functions. This is a closed subspace, hence for
every f ∈ L2 the perpendicular projection g0 = πU f is well defined. We claim that

f (x) + f (−x)
g0 (x) = .
2

36
Indeed, g0 (x) = g0 (−x) for every x ∈ R, hence g0 ∈ U . It remains to check that f − g0 is
perpendicular to every function g ∈ U . This is indeed the case because, if g is even,
Z   Z Z
f (x) + f (−x) f (x) f (−x)
g(x) f (x) − dx = g(x) dx − g(x) dx
2 2 2
Z Z
f (x) f (x)
= g(x) dx − g(−x) dx = 0 .
2 2

16. To prove that (i)=⇒(ii), assume that K is compact. Let B1 = {x ∈ X ; kxk ≤ 1} be the
closed unit ball in X. Given any ε > 0, by assumption the image K(BS1 ) is precompact, hence
it can be covered with finitely many balls of radius ε. Say, K(B1 ) ⊆ ni=1 B(yi , ε).
.
Call Y = span{y1 , . . . , yn } and set
.
Kε (x) = πY ◦ K(x) .

Clearly, Kε has finite dimensional range. Moreover,

kK − Kε k = sup kK(x) − Kε (x)k = sup kK(x) − πY K(x)k ≤ sup d(K(x), Y ) ≤ ε .


kxk≤1 kxk≤1 kxk≤1

Indeed,
d(K(x), Y ) ≤ min kK(x) − yi k ≤ ε .
1≤i≤n

The converse implication (ii)=⇒ (i) is an immediate consequence of Theorem 4.10.

P
17. Assume that n kun − vn k < 1. Let (un )n≥1 be complete. If (vn )n≥1 is not complete,
then there exists a unit vector x ∈ H which is perpendicular to every vn . A contradiction is
obtained by writing

X ∞
X ∞
X ∞
X
2 2 2 2 2
1 = kxk = (x, un ) = (x, un − vn ) ≤ kxk kun − vn k ≤ kun − vn k .
n=1 n=1 n=1 n=1

18. (i) Let (un )n≥1 be mutually orthogonal. Assume θ0 y + θ1 u1 + · · · + θN uN = 0, for some
coefficients θj not all zero. Since the vectors u1 , . . . , uN , uN +1 are linearly independent, we
must have θ0 6= 0 and uj 6= 0 for every j. This yields a contradiction because

0 = (0, uN +1 ) = (θ0 y + θ1 u1 + · · · + θN uN , uN +1 ) = θ0 (y, uN +1 ) = θ0 2−N −1 kuN +1 k2 .

. P .
(ii) Let (un )n≥2 be an orthonormal set, and define v = ∞n=2 2
−n u , u =
n 1 v/kvk. Observe
that in this case the two vectors y, u1 are parallel.

19. Assume that the strong convergence kxn − xk → 0 holds. Then for every y ∈ H one has

|(y, xn ) − (y, x)| ≤ kyk · kxn − xk → 0 as n → ∞.

37
Hence one has the weak convergence xn * x as well. Moreover, if kxn k ≥ kxk, then

kxn k − kxk ≤ kxn − xk → 0 as n → ∞.

Viceversa, assume xn * x and kxn k → kxk. Then

lim sup kxn − xk2 = lim sup (xn − x , xn − x)


n→∞ n→∞
 
= limn→∞ (xn , xn ) − (xn , x) − (x, xn ) + (x, x) = kxk2 − kxk2 − kxk2 + kxk2 = 0.

 Z 1 
2
20. The orthogonal subspace is U⊥ = W = w ∈ L (Q) ; w(x, y) dx = 0 for a.e. y ∈ [0, 1] .
0

Given f ∈ L2 (Q), we have the perpendicular decomposition f = u + w with u ∈ U , w ∈ W =


U ⊥ . Here
Z 1 Z 1
u(x, y) = ϕ(y) = f (s, y) ds , w(x, y) = f (x, y) − f (s, y) ds .
0 0

Observe that u and w are perpendicular because


Z Z 1 Z 1  Z 1
u(x, y)w(x, y) dxdy = ϕ(y) w(x, y) dx dy = ϕ(y) · 0 dy = 0 .
Q 0 0 0

In this case, the function g ∈ U which has minimum distance from f is the perpendicular
projection: g = πU (f ) = u.

21. Repeat the arguments in the proof of Theorem 4.2.

22. (i) The closure and convexity of Ω ⊂ L2 (R) are straightforward. If kfn − f kL2 → 0, we
can find a subsequence such that fn (x) → f (x) for a.e. x ∈ R. If fn (x) ≤ ex for every n ≥ 1
and a.e. x ∈ R, then also f (x) ≤ ex for a.e. x. Hence Ω is a closed subset of L2 (R).
Moreover, if f (x) ≤ ex and g(x) ≤ ex for a.e. x ∈ R, the same is true for any convex
combination: θf (x) + (1 − θ)g(x) ≤ ex for every θ ∈ [0, 1] and a.e. x.
(ii) Clearly the function g(x) = min{f (x), ex } lies in the convex set Ω. By the result proved
in problem 21, it suffices to check that
Z
(ω(x) − g(x)) · (g(x) − f (x)) dx ≥ 0 for all ω ∈ Ω . (14)

By assumption, ω(x) ≤ ex for a.e. x. For each x ∈ R we consider two cases.


Case 1: f (x) > ex . Then g(x) = ex and therefore

ω(x) − g(x) ≤ 0 , g(x) − f (x) < 0 .

Case 2: f (x) ≤ ex . Then g(x) = f (x) and therefore g(x) − f (x) = 0.

38
In both cases, (ω(x) − g(x)) · (g(x) − f (x)) ≥ 0. Hence (14) holds.


23. (i) The operator Λ has norm kΛk = 2. Indeed,
Z 0 Z ∞ 1/2  Z ∞ 1/2
kΛf kL2 = |f (−x)|2 dx + |f (x)|2 dx = 2 |f (x)|2 dx
−∞ 0 0

 1/2 √
≤ 2kf k2L2 = 2 · kf kL2 ,

with equality holding whenever f is supported on the positive half axis.


(ii) Ker(Λ) = {f ∈ L2 ; f (x) = 0 for a.e. x > 0},
Range(Λ) = {f ∈ L2 ; f (x) = f (−x) for a.e. x ∈ R}.
(iii) The adjoint operator Λ∗ is defined by the identity
Z Z Z 0 Z ∞
f (x)(Λ∗ g)(x) dx = (Λf )(x)g(x) = f (−x)g(x) dx + f (x)g(x)dx
−∞ 0
Z ∞  
= f (x) g(x) + g(−x) dx .
0

This implies 
0 if x < 0,
(Λ∗ g)(x) =
g(−x) + g(x) if x > 0.

24. (i) For any f ∈ L2 ([0, ∞[), the substitution y = ex yields


Z ∞ Z ∞ Z ∞
x 2 |f (y)|2
|f (e )| dx = dy ≤ |f (y)|2 dy .
0 1 y 0

Hence kΛf kL2 ≤ kf kL2 and kΛk ≤ 1. To prove the converse inequality consider the sequence

of functions fn = n · χ[1,1+1/n] . Then
Z 1+1/n
2 n  1
kfn kL2 = 1 , kΛfn k = dy = n ln 1 + → 1 as n → ∞.
1 y n
Therefore kΛk = 1.
n o
(ii) Ker(Λ) = f ∈ L2 ([0, ∞[) ; f (x) = 0 for a.e. x ∈ [0, 1] .
Z ∞
n
2 |g(ln y)|2 o
Range(Λ) = g ∈ L ([0, ∞[) ; dy < ∞ .
1 y
(iii) The adjoint operator Λ∗ is characterized by the identity
Z ∞ Z ∞ Z ∞ Z ∞
g(ln y)
f (x)(Λ∗ g)(x) dx = (Λf )(x)g(x) dx = f (ex )g(x) dx = f (y) dy .
0 0 0 1 y
Therefore 

 0 if y ∈ [0, 1] ,


(Λ g)(y) =
g(ln y)
if y > 1 .



y

39
25. Assume fn * f . Then for any b ∈ [0, T ], taking g = χ[0,b] ∈ L2 ([0, T ]) we have
Z b Z T Z T Z b
lim fn (x) dx = lim fn (x)g(x) dx = f (x)g(x) dx = f (x) dx .
n→∞ 0 n→∞ 0 0 0

Viceversa, assume that kf kL2 ≤ M , kfn kL2 ≤ M for all n ≥ 1, and that
Z b Z b
lim fn (x) dx = f (x) dx for every b ∈ [0, T ] .
n→∞ 0 0

If ϕ is a piecewise constant function of the form


N
X
ϕ = ck · χ[0,bk ] (15)
k=1

for some constants ck , bk , then by linearity we still have


Z T N
X Z bk N Z
X bk Z T
lim fn (x)ϕ(x) dx = lim ck fn (x) dx = ck f (x) dx = f (x)ϕ(x) dx .
n→∞ 0 n→∞ 0 0 0
k=1 k=1

Now consider an arbitrary function g ∈ L2 ([0, T ]). Given any ε > 0, we can find a piecewise
constant function ϕ of the form (15) such that kg − ϕkL2 < ε. This yields the estimate
Z T Z T Z T

lim sup
(fn − f )g dx ≤ lim sup
(fn − f )ϕ dx + sup
(fn − f )(ϕ − g) dx
n→∞ 0 n→∞ 0 n≥1 0

≤ 0 + kfn − f kL2 kϕ − gkL2 ≤ 2M ε .


RT RT
Since ε > 0 is arbitrary, this implies 0 fn g dx → 0 f g dx, for every g ∈ L2 ([0, T ], proving
the weak convergence fn * f .


26. Consider first the case fn (x) = n · cos nx . Then for any b ∈ [0, 1] we have
Z b
1
fn (x) dx = √ sin nb → 0 as n → ∞ .
0 n
By taking linear combinations, it is clear that
Z 1
lim fn (x)g(x) dx = 0 (16)
n→∞ 0

for any piecewise constant function g. In spite of the fact that piecewise constant functions
are dense in L2 ([0, 1]), the weak convergence fn * 0 FAILS, because the sequence (fn )n≥1 is
not bounded. Indeed,
Z 1 Z 1
2
|fn (x)| dx = n cos2 nx dx → ∞ as n → ∞ .
0 0

Next, consider the case

n2/3 if x ∈ [0, n−1 ] ,



fn (x) =
0 if x > n−1 .

40
For each fixed 0 < b ≤ 1, and all n > b−1 we have
Z b Z 1/n
fn (x) dx = n2/3 dx = n−1/3 → 0 as n → ∞ .
0 0

Hence, also in this case (16) holds for every piecewise constant function g. However,
Z 1 Z 1/n
2
|fn (x)| dx = |n2/3 |2 dx = n1/3 → ∞ as n → ∞ .
0 0

Also in this case, the sequence (fn )n≥1 is not bounded in L2 ([0, 1]), hence it cannot converge
weakly. Notice that in this case the result proved in problem 25 cannot be used.
For example, take g(x) = x−3/7 . Then g ∈ L2 ([0, 1]) but
Z 1 Z 1/n
7
fn (x)g(x) dx = n2/3 x−3/7 dx = n2/3 · n−4/7 → ∞ as n → ∞ .
0 0 4

27. We only need to prove the implication (ii) =⇒ (i). By linearity, it is clear that the
convergence (xn , y) → (x, y) must hold for every y ∈ span(S). By assumption, span(S) is a
subspace whose closure has non-empty interior. Hence span(S) = H.
Let y ∈ H be given. For any ε > 0 we can find a point ỹ ∈ span(S) with ky − ỹk < ε. By
assumption, there exists a constant M such that kxk ≤ M and kxn k ≤ M for all n ≥ 1. This
yields the estimate

lim sup (xn − x, y) ≤ lim sup (xn − x, y − ỹ) + lim sup (xn − x, ỹ)

n→∞ n→∞ n→∞

≤ sup kxn − xk · ky − ỹk + 0 ≤ 2M · ε .


n≥1

Since ε > 0 was arbitrary, this proves the weak convergence xn * x.

28. Use the result in problem 27, choosing S as the set of all characteristic functions χQ ,
where Q = [a1 , b1 ] × · · · × [aN , bN ]. Observe that the subspace spanned by all these functions
is dense on L2 (Ω).

29. If y ∈
/ S = co{xn ; n ≥ 1}, then by Theorem 2.33 there exists a linear functional φ : H 7→
R that strictly separates the compact, closed convex set {y} from the closed convex set S. By
the Riesz’ representation theorem, there exists an element z ∈ H and constants c1 < c2 such
that
(y, z) = φ(y) ≤ c1 < c2 ≤ φ(xn ) = (xn , z) for all n ≥ 1 .
This leads to a contradiction, because the weak convergence xn * y implies (xn , z) → (y, z).

Chapter 6

41
1. (i) Define

. f (x − 1) if x > 1 ,
(Λ1 f )(x) = f (x + 1) , (Λ2 f )(x) =
0 if x ∈ [0, 1] .

Then 
f (x) if x > 1 ,
(Λ1 ◦ Λ2 f )(x) = f (x) , (Λ2 ◦ Λ1 f )(x)
0 if x ∈ [0, 1] .

(ii) If Λ(I − K) = I, then Ker(I − K) = {0} hence the Fredholm operator I − K is one-to-one.
By Theorem 6.1, I −K is onto. We conclude that I −K is a bijection, and hence Λ = (I −K)−1
commutes with I − K.
On the other hand, if (I − K)Λ = I, then I − K must be onto. Hence by Theorem 6.1 the
Fredholm operator I − K is one-to-one. We conclude that I − K is a bijection, and hence
Λ = (I − K)−1 commutes with I − K.

2. (i) Λ is a bounded operator, with norm kΛk = 2. Indeed,


Z ∞ Z ∞ Z ∞
2 2 2
kΛf kL2 ([0,∞[) = |(Λf )(x)| dx = |2f (x+1)| dx = 4 |f (y)|2 dy ≤ 4kf kL2 ([0,∞[) ,
0 0 1

with equality holding if f (x) = 0 for a.e. x ∈ [0, 1].


Λ is not compact. Indeed, consider the sequence of characteristic functions fn = χ .
[n, n+1]
Then √
kfn kL2 = 1 , kΛfn − Λfm kL2 = 2 2 for all m 6= n .
Therefore, from the sequence (Λfn )n≥1 one cannot extract any convergent subsequence.
(ii) To compute the adjoint operator we write
Z ∞ Z ∞ Z ∞ Z ∞
f (y)(Λ∗ g)(y) dy = (Λf )(x)g(x) dx = 2f (x+1)g(x) dx = f (y) 2g(y−1) dy.
0 0 0 1

Therefore 
∗ 2g(y − 1) if y > 1 ,
(Λ g)(y) =
0 if y ∈ [0, 1] .
n o
(iii) One has KerΛ = f ∈ L2 ([0, ∞[ ; f (x) = 0 for a.e. x ∈ [0, 1] , while KerΛ∗ = {0}.

Of course, this implies that Λ is not a Fredholm operator. In particular, it cannot be written
as λI + K, with λ ∈ R and K compact.

3. Assume η > M . Then the operator ηI − Λ is strictly positive definite. Indeed,

(ηx − Λx , x) ≥ ηkxk2 − kΛxkkxk ≥ (η − kΛk) kxk2 .

An application of Theorem 5.12, with β = η − kΛk, yields the existence of a continuous inverse
(ηI − Λ)−1 , having norm k(ηI − Λ)−1 k ≤ 1/β. Therefore, η ∈ σ(Λ).
The case η < −M is entirely similar.

42
.
4. (i) Let M = max(x,y)∈[0,1]×[0,1] |K(x, y)| . Then, by Hölder’s inequality,
Z 1 Z 1

|Λf (x)| =
K(x, y)f (y) dy ≤ M
|f (y)| dy ≤ M kf kL2 ([0,1]) .
0 0

This proves that Λ is bounded, with kΛk ≤ 1.


Since K(x, y) = K(y, x), the fact that Λ is self-adjoint follows from
Z 1 Z 1  Z 1Z 1
(Λf, g)L2 = K(x, y)f (y) dy g(x) dx = K(x, y)f (y)g(x) dxdy
0 0 0 0

Z 1 Z 1 
= K(y, x)g(x)dx f (y)dy = (f, Λg)L2 .
0 0

To show that Λ is compact, follow the proof of Theorem 4.12. Observe that, if a sequence
of continuous functions gn = Λfn converges uniformly on [0, 1], then it also converges in
L2 ([0, 1]).
(ii) Consider the special case where
Z x Z 1
u(x) = (Λf )(x) = (1 − x)y f (y) dy + (1 − y)xf (y) dy .
0 x
Differentiating twice w.r.t. x one obtains
Z x Z 1
0
u (x) = (−y)f (y) dy + (1 − y)f (y) dy ,
0 x
00
u (x) = − f (x) .

5. (i) The fact that Λ is self-adjoint is checked by writing


Z π Z π Z π
f (x) (Λ∗ g)(x) dx = (Λf )(x) g(x) dx = f (x) sin x g(x) dx .
0 0 0

Hence (Λ∗ g)(x) = sin x g(x).


(ii) For every f ∈ L2 ([0, π]) one has
Z π 2 Z π 2
2
kΛf kL2 = sin x f (x) dx ≤ f (x) dx .

0 0
n
Hence kΛk ≤ 1. To prove the converse inequality, consider the set An = x ∈ [0, π] ; sin x >
o  
1− n1 and define fn = χAn . Then kΛfn kL2 ≥ 1− n1 kfn kL2 . We thus conclude that kΛk = 1.
However, there is no function f ∈ L2 such that Λf = f . Hence 1 ∈
/ σp (Λ).
(iii) According to problem 20 in Chapter 4, the multiplication operator is not compact.

6. (i) Assume that u ∈ L2 ([0, 1]), say with kukL2 = M . Then, for any 0 ≤ t1 < t2 ≤ 1, using
Hölder’s inequality we obtain
Z t2

Λu(t2 ) − Λu(t1 ) ≤ 1 · |u(s)| ds ≤ k1kL2 ([t1 ,t2 ]) · kukL2 = t2 − t1 · kukL2 .

t1

43
Hence Λu ∈ C 0,1/2 .
(ii) Consider any sequence (fn )n≥1 with kfn kL2 ≤ 1 for every n. Then the corresponding func-
tions Λfn are uniformly bounded and Hölder continuous, hence equicontinuous. By Ascoli’s
theorem, one can extract a subsequence (fnk )k≥1 that converges uniformly on [0, 1] to some
continuous function f . In turn, this implies kfnk − f kL2 → 0, proving that the operator Λ is
compact.
(iii) Writing
Z 1 Z 1 Z 1 Z x 

u(x) (Λ v)(x) dx = (Λu)(x) v(x) dx = u(s) ds v(x) dx
0 0 0 0
Z 1  Z 1  Z 1 Z x 
= u(s) ds v(s) ds − u(x) v(s) ds dx ,
0 0 0 0

we conclude Z 1

(Λ v)(x) = v(s) ds .
x
Rx
(iv) The operator (I−K) is a Fredholm operator on L2 ([0, 1]). If u = Ku, then u(x) = 0 u(s) ds .
hence u is an absolutely continuous solution of the Cauchy problem
d
u(x) = u(x) , u(0) = 0 .
dx
By Gronwall’s inequality, the only solution is u(x) = 0 for every x. We conclude that the
operator I − K is one-to-one. By Fredholm’s theorem, I − K is onto. Hence for every
g ∈ L2 ([0, 1]) there exists one and only one function u such that u − Ku = g. The inverse
map g 7→ u = (I − K)−1 is also a bounded linear operator.
If g is continuously differentiable, then u is an absolutely continuous function that satisfies
the Cauchy problem

u0 (x) = g 0 (x) + u(x) , u(0) = g(0) .

To compute the solution u(·), we set v = u − g. The previous ODE yields

v 0 (x) = v + g , v(0) = 0 ,
Z x
v(x) = ex−y g(y) dy .
0
This provides an explicit formula for the inverse operator:
  Z x
−1
u(x) = (I − K) g (x) = g(x) + ex−y g(y) dy .
0

7. To fix the ideas, let w1 , . . . , wN be the eigenvectors with corresponding eigenvalues λ1 =


· · · = λN = 1, while λk 6= 1 for k > N . Then the equation u − Ku = f admits a solution if
and only if
(f, wk ) = 0 for every k ∈ {1, . . . , N }. (17)

44
If (17) holds, then u is a solution if and only if
N
X X (f, wk )
u = ck wk + wk ,
1−λ
k=1 k>N

for arbitrary constants c1 , . . . , cN .

8. Assume A = A∗ , B = B ∗ . Then for every x, y ∈ H we have

(ABx, y) = (Bx, Ay) = (x, BAy).

Hence AB is self adjoint if and only AB = BA.

9. In the case where H is finite dimensional, the result is trivial: every operator is compact
and (ii) is satisfied simply because the set of orthonormal sequences is empty. In the following
we thus assume that H is infinite dimensional.
(i) =⇒ (ii). Assume that Λ : H 7→ H is compact, and let {v1 , v2 , . . .} be an orthonormal
sequence of vectors in H.
If the sequence Λvn does not converge to zero, by taking a subsequence we can assume kΛvn k ≥
ε > 0 for all n ≥ 1. A contradiction is obtained as follows.
By compactness, there exists a subsequence such that Λvnk → w for some vector w. Our
previous assumption implies kwk ≥ ε.
By choosing a further subsequence, we can assume
kwk
kΛvnk − wk < for all k ≥ 1 . (18)
2

Consider the vector v = ∞


P
k=1 vnP
k
/k. Since the unit vectors vnk are mutually orthogonal, the
series is convergent and kvk2 = ∞ k=1 k
−2 < ∞. Since Λ is a bounded operator, we should

have

N N
!
X Λvn k
X vn k
= Λ → Λ(v) as N → ∞.
k k
k=1 k=1

But this is impossible, because by (18)



N N N N  
X Λv
nk
X w X w − Λvnk X 1 kwk
≥ − ≥ kwk −

k k k k 2


k=1 k=1 k=1 k=1

and the right hand side approaches infinity as n → ∞.


.
(ii) =⇒ (i). Assume that Λ is not compact. Then the set S = Λ(B1 ) defined as the closure
of the image of the unit ball, is not compact. By the result proved in problem 15, Chapter 4,
there exists ε > 0 such that

(Pε ) S is not contained in the ε-neighborhood of any finite-dimensional subspace of H.

45
We now inductively construct an orthonormal sequence {v1 , v2 , . . .}, as follows.

• By (Pε ), there exists a vector v1 such that kΛv1 k ≥ ε.

• Assume that the orthonormal set {v1 , . . . , vn } has been constructed. Again by (Pε ),
.
the set S is not contained in the ε-neighborhood of the finite dimensional space Vn =
span{Λv1 , . . . , Λvn }. Hence there exists a vector w such that kwk ≤ 1 and d(Λw, Vn ) ≥ ε.
Define the vectors
n
. X . wn+1
wn+1 = w− (w, vn )vn , vn+1 = .
kwn+1 k
k=1

Observe that wn+1 is the perpendicular projection of w on span{v1 , . . . , vn }⊥ . Hence


0 < kwn+1 k ≤ kwk ≤ 1. Moreover, observing that Λwn+1 − Λw ∈ Vn , we obtain

kΛvn+1 k ≥ kΛwn+1 k ≥ d(Λwn+1 , Vn ) = d(Λw, Vn ) ≥ ε .

By induction, we thus obtain an orthonormal sequence {vn ; n ≥ 1} such that kΛvn k ≥ ε > 0
for every n. Hence (ii) fails.

To prove the last statement, consider the orthonormal sequence


X
wn = 2−n/2 ek .
2n <k≤2n+1

Then define the operator Λ by setting



. X
Λv = (wn , v)en .
n=1

This is a bounded linear operator with norm kΛk = 1. It is not compact, because the
image of the unit ball contains the entire orthonormal sequence {en ; n ≥ 1}. However,
limk→∞ Λek = 0. Indeed, for 2n < k ≤ 2n+1 we have

kΛek k = |(wn , ek )| = 2−n/2 .

10. Let V ⊆ H be a closed subspace. Consider the orthogonal decomposition V = V1 + V2 ,


where
. .
V1 = V ∩ Ker(I − K) , V2 = V ∩ V1⊥ .
Then (I − K)(V ) = (I − K)(V2 ). By possibly replacing V with the closed subspace V2 , we
can thus assume that V ∩ Ker(I − K) = {0}, i.e. I − K is one-to-one restricted to V .
We claim that there exists a constant β > 0 such that

ku − Kuk ≥ β kuk for all u ∈ V. (19)

Otherwise, we could find a sequence (un )n≥1 such that


.
kun k = 1 , yn = un − Kun → 0 .

46
Since K is compact, there exists a subsequence such that Kunj → z for some z ∈ H. This
yields
lim un = lim (yn + Kun ) = z ∈ V .
n→∞ n→∞
Hence
(I − K)z = lim (I − K)un = 0 ,
n→∞
in contradiction with the assumption that (I − K) is one-to-one restricted to V .
Now assume vn ∈ V for every n ≥ 1, and

yn = (I − K)vn → y.

We need to prove that y ∈ (I − K)(V ).


Using (19) we obtain

0 = lim sup kym − yn k ≥ β lim sup kvm − vn k .


m,n→∞ m,n→∞

Therefore the sequence (vn )n≥1 is Cauchy, and converges to some limit v ∈ V . By continuity,
y = (I − K)v.

11. Let {w1 , w2 , . . .} be an orthonormal basis of a real Hilbert space H, consisting of eigen-
vectors of a linear, compact, self-adjoint operator K. Let λ1 , λ2 , . . . be the corresponding
eigenvalues.
Assume that t 7→ u(t) provides a solution to the Cauchy problem
d
u(t) = Ku(t) , u(0) = f , (20)
dt
for some f ∈ H. Taking the inner product of u(t) with wk we obtain
d
(u(t), wk ) = (Ku(t), wk ) = (u(t), Kwk ) = λk (u(t), wk ) , (u(0), wk ) = (f, wk ) .
dt
(21)
Therefore, the solution can be written as

X
u(t) = ck (t)wk , (22)
k=1

where each coefficient ck (·) is obtained by solving the linear scalar Cauchy problem

c0k (t) = λk ck (t) , ck (0) = (f, wk ) .

This yields ck (t) = eλk t (f, wk ), and hence


X
u(t) = eλk t (f, wk ) wk .
k≥1

12. In the more general case of a non-homogenous equation, the coefficients ck (·) satisfy the
equations
c0k (t) = λk ck (t) + (g(t), wk ) , ck (0) = (f, wk ) .

47
Hence the formula (22) remains valid, with
Z t
ck (t) = eλk t (f, wk ) + eλk (t−s) (g(t), wk ) ds .
0

13. Taking the inner product of the solution with wk , in this case we find that the function
.
ck (t) = (u(t), wk ) satisfies the second order initial value problem

c00k (t) = λk ck (t) , ck (0) = (f, wk ) , c0k (0) = (g, wk ) .

14. Assume that ηI − Λ is a continuous bijection. Then there exists β > 0 such that, for
every bounded linear operator Ψ with norm kΨk < β, the operator ηI − Λ + Ψ is a continuous
bijection. Hence every η̃ ∈ R with |η̃ − η| < β lies in the resolvent set ρ(Λ).

Chapter 7

1. (i) The norm of the diagonal operator ASt is computed by



kASt k = sup λk eλk t .

k

Using the assumption

λk = αk + iβk = ω − rk (cos θk + i sin θk )

for some rk ≥ 0 and |θk | ≤ θ̄ < π/2, we obtain


.
|λk eλk t | ≤ (|ω| + rk )e(ω−rk cos θk )t ≤ (|ω| + rk )e(ω−ηrk )t η = cos θ̄ > 0 .

Hence, for a fixed t > 0,

kASt k ≤ sup (|ω| + r)e(ω−ηr)t < ∞ .


r≥0

(ii) Similarly,

kAn St k = sup λnk eλk t ≤ sup (|ω| + r)n e(ω−ηr)t < ∞ .

k r≥0

2. This is an immediate consequence of the formula (7.31). Namely, if S is a semigroup of


type ω, then
ketAλ k ≤ e2ωt
for all t ≥ 0 and λ ≥ 2ω. If S is contractive semigroup, then S is of type ω = 0.

48
3. The computation of the various integrals is an exercise in basic Calculus.
From the identities
Z ∞ ∞ ∞
T2
Z Z
Wn (t) dt = 1 , tWn (t) = T , (t − T )2 Wn (t) dt =
0 0 0 n
we deduce r
T2
Z
Wn (t) dt ≤ → 0 as n → ∞.
|t−T |>ε n
This proves (7.39).

4. The formula Z ∞
−n
(I − hA) u = wn (t)St u dt (23)
0
is proved by induction on n. The case n = 1 is already known. Now assume (23) is valid.
Using the variable τ = t + s we obtain
Z ∞  
−(n+1) −1 −n
(I − hA) u = (I − hA) (I − hA) u = w(t)St (I − hA)−n u dt
0
Z ∞ Z ∞  Z ∞
= w(t)St wn (s)Ss u ds dt = (w ∗ wn )(τ )Sr u dτ .
0 0 0

Hence the same formula (23) is valid with n replaced by n + 1.


The convergence
 −n Z ∞
T
I− A u = Wn (t)St u dt → ST u as n → ∞. (24)
n 0

is proved by using (7.39). Indeed, let ε > 0 By the continuity of the map t 7→ St u there exists
δ > 0 such that

kSt u − ST uk ≤ ε for all t ∈ [T − δ, T + δ] .

By (7.39), for all n large enough we have


Z
Wn (t) dt < ε .
|t−T |>δ

Recalling that kSt uk ≤ kuk for every t ≥ 0, we thus obtain


 −n Z ∞
T
ST u − I − A u ≤ ST u − Wn (t)St u dt

n 0


Z Z Z

≤ S
T u − Wn (t)ST u dt
+ Wn (t)kSt u − ST uk dt + Wn (t)kSt u − ST uk dt
0 |t−T |≤δ |t−T |>δ

≤ 0 + ε + ε 2kuk .

Since ε > 0 was arbitrary, this proves (24).

49
5. (i) =⇒ (ii). Assume that St u ∈ Ω for every u ∈ Ω and t ≥ 0. Then

e−t/h
Z
−1
(I − hA) u = St u dt .
0 h
−t/h
This showsRthat (I −hA)−1 u is an integral average of points St u ∈ Ω, with weight w(t) = e h

satisfying 0 w(t) dt = 1. Approximating the integral with a finite sum, we see that (I −
hA)−1 u can be approximated by convex combinations of elements Sti u ∈ Ω. Since Ω is closed
and convex, we conclude (I − hA)−1 u ∈ Ω.

(ii) =⇒ (i). Using the result of the previous problem 4,


 −n Z ∞
T
I− A = Wn (t)St u dt , (25)
n 0
R∞
where Wn is a smooth averaging kernel, with 0 Wn (t) dt = 1. Since Ω is closed and convex,
the right hand side of (25) lies in Ω. Letting n → ∞, by the previous problem 4 the left hand
side of (25) converges to ST u. Since Ω is closed, we conclude ST u ∈ Ω.

6. Let {St ; t ≥ 0} be a semigroup of type ω, with generator A. Then

keγt St uk = eγt kSt uk ≤ eγt eωt kuk .

Hence {eγt St ; t ≥ 0} is a semigroup of type γ + ω. Moreover, if u ∈ Dom(A), then

eγh Sh u − u eγh Sh u − Sh u Sh u − u
lim = lim + lim = γu + Au .
h→0+ h h→0+ h h→0+ h

Calling Aγ the generator of the semigroup {eγt St ; t ≥ 0}, this proves that Dom(Aγ ) ⊇
Dom(A) and Aγ (u) = Au + γu.
Inverting the roles of A, Aγ , we see that Dom(A) ⊇ Dom(Aγ ) and Au = Aγ u − γu. This
concludes the proof.

7. (i) The semigroup properties are easily checked:

(S0 f )(x) = e−2·0 f (x + 0) = f (x) ,


 
(St+s f )(x) = e−2(t+s) f (x + t + s) = e−2s e−2t f ((x + t) + s) = (Ss (St f ))(x) .

In the following, we first assume 1 ≤ p < ∞. By definition, the function f ∈ Lp (R) lies in the
domain of the generator A if and only if the following limit exists in Lp (R):

. e−2h f (x + h) − f (x) e−2h f (x + h) − f (x + h) f (x + h) − f (x)


Au = lim = lim + lim
h→0+ h h→0+ h h→0+ h

f (x + h) − f (x)
= − 2f (x) + lim .
h→0+ h
(26)

50
Hence
n o
Dom(A) = u ∈ Lp (R) ; u is absolutely continuous and ux ∈ Lp (R)

Moreover, Au = −2u + ux for every u ∈ Dom(A).


(ii) On the space L∞ (R) the above semigroup is not strongly continuous. For example, consider
the function 
1 if x ∈ [0, 1], ,
u(x) =
0 otherwise .
Then
e−2t

if x ∈ [−t, 1 − t] ,
St u(x) =
0 otherwise .
As t → 0+ we then have the convergence kSt u − ukLp (R) → 0 for every 1 ≤ p < ∞. However

lim kSt u − ukL∞ (R) = 1 .


t→0+

Hence the map t 7→ St u is not continuous from [0, ∞[ into the space L∞ (R).

8. The generator A of the semigroup must be a linear operator on Rn with dense domain.
The only dense subspace of Rn is Rn itself. Hence Dom(A) = Rn , and A must be described
by an n × n matrix. In particular, the linear operator A is continuous, hence for every u ∈ Rn
and t ≥ 0 we must have
d
St u = ASt u, S0 u = 0 .
dt
Since t 7→ St u is the solution to the above Cauchy problem, we conclude that St u = etA u.

9. (i) On the space of all bounded continuous functions w : [0, T ] 7→ X, consider the Picard
operator defined as Z t
.
Φ(w)(t) = St ū + St−s f (s, w(s)) ds .
0
Following the proof of Theorem 7.1, one checks that Φ is a strict contraction w.r.t. the equiv-
alent norm
.
kwk† = max e−2Lt kw(t)k .
t∈[0,T ]

Hence Ψ has a unique fixed point, which by definition provides a mild solution of (7.40).
(ii) Assume that the generator A is a bounded linear operator. Then we can differentiate
(7.41) w.r.t. time, and obtain

d+ St+h ū − St ū 1 t+h
Z
u(t) = lim + lim St−s f (s, u(s)) ds
dt h→0+ h h→0+ h t

t
St+h−s f (s, u(s)) − St−s f (s, u(s))
Z
+ lim ds
h→0+ 0 h
Rt
= ASt ū + f (t, u(t)) + 0 ASt−s f (s, u(s)) ds

= Au(t) + f (t, u(t)) .

51
Observing that the forward derivative (obtained by taking the limit as h → 0+) is a continuous
function of time, we conclude that it coincides with the backward derivative (obtained by
letting h → 0−). This completes the proof.

10. For u ∈ L1 ([0, 1]), define




 0 if x ≤ t/T ,
(St u)(x) = (27)
 u x− t
 
if x > t/T .

T
This corresponds to solving the PDE ut + ux = 0 with boundary condition u(t, 0) = 0.

11. Assume Sτ is compact. If t > τ then St = St−τ Sτ is the composition of the continuous
operator St − τ with the compact operator Sτ , hence it is compact.
To see that the converse may not hold in general, observe that the semigroup (27) is trivially
compact for t ≥ T but not compact for 0 ≤ t < T .

12. (i) (St u)(x) = u(x + t). Not that this is well defined also for t < 0. This is a group of
isometries: kSt ukL1 = kukL1 .
.
(ii) If v = Eh− u = (I −hA)−1 u, then v = u+hvx . We are thus looking for a function v ∈ L1 (R)
such that v − hv 0 = u, hence
v(x) u(x)
v 0 (x) = − x ∈ R.
h h
The explicit solution is provided by

e(x−y)/h
Z
v(x) = u(y) dy.
x h
Setting t = y − x, it is interesting to observe that the above formula is equivalent to
Z ∞ −t/h Z ∞ −t/h
e e
v(x) = u(x + t) dt = (St u)(x) dt .
0 h 0 h

(iii) If u ∈ Cc∞ with support contained in the interval [a, b], then the same is true of the
derivative ux . Hence (Eh+ u)(x) = u(x) + hux (h) is a smooth function supported inside [a, b].
By induction on n we see that the same holds for (Eh+ )n u.
(iv) Take T ≥ b − a. If u vanishes outside [a, b], then ST u is supported inside [a − T, b − T ].
In particular, (ST u)(x) = u(x + T ) = 0 for every x ∈ [a, ∞[ .
On the other hand, every forward Euler approximation vanishes outside the interval [a, b],
hence the same must be true for the limit (if it exists). We conclude that, if the function u is
not identically zero, the forward Euler approximations cannot converge to ST u.

Chapter 8

52
1. (i) This is a distribution of infinite order.
(ii)-(iii) These linear functionals are not distributions on R. They are both distributions on
the open half line ]0, ∞[ .
(iv) This is a distribution of order zero on Ω = ]0, ∞[ .

2. By assumption, f has a weak derivative Df ∈ Lp (]a, b[) ⊂ L1 (]a, b[). By Corollary 8.17, f
coincides a.e. with an absolutely continuous function.
If f is continuously differentiable, for any a < x < y < b, using Hölder’s inequality on the
interval [x, y] we obtain
Z y
1 1
|f (x) − f (y)| ≤ 1 · |f 0 (s)| ds ≤ k1kLq ([x,y]) · kf 0 kLp ([x,y]) + = 1.
x q p
1− p1
k1kLq ([x,y]) = |x − y|1/q = |x − y| , kf 0 kLp ([x,y]) ≤ kf 0 kLp ([x,y]) ≤ kf kW 1,p (]a,b[) .
By approximation, the same inequality holds for every f ∈ W 1,p .

3. Fix any δ > 0 and consider the smaller square


.
n o
Qδ = (x1 , x2 ) ; δ < x1 < 1 − δ, δ < x2 < 1 − δ .
.
For any ε ∈ ]0, δ], the mollified functions fε = Jε ∗ f are well defined on Qδ . Moreover
Dx1 fε = Jε ∗ Dx1 f = 0. Hence there exist a smooth function gε : [ε, 1 − ε] 7→ R such that
fε (x1 , x2 ) = gε (x2 ) for all (x1 , x2 ) ∈ Qδ .
Letting εn → 0, we obtain kfε − f kL1 (Qδ ) → 0. Hence, for a suitable sequence εn → 0 we
achieve the pointwise convergence

f (x1 , x2 ) = lim fε (x1 , x2 ) = lim gε (x2 ) for a.e. (x1 , x2 ) ∈ Qδ .


ε→0 ε→0

Since δ > 0 is arbitrary, this proves the result.

4. For every test function ϕ ∈ Cc∞ (Ω0 ) we have


Z Z Z
∂f
g ϕ dx = − f Dx1 ϕ dx = ϕ dx .
Ω0 Ω0 Ω0 ∂x1
∂f
By the uniqueness of the weak derivative, proved in Lemma 8.12, this implies g(x) = ∂x1 (x)
for a.e. x ∈ Ω0 .

.
5. (i) Consider the mollifications uε = Jε ∗ u. The assumption u ∈ W 1,∞ (Ω) implies that
.
|∇uε (x)| ≤ kukW 1,∞ for all x ∈ Ωε = {x ∈ Ω ; B(x, ε) ⊂ Ω} .

Since Ωε is convex, for any x, y ∈ Ωε we have


Z 1  
|uε (x)−uε (y)| ≤ |∇uε (θx+(1−θ)y)·(x−y)| dθ ≤ max |∇uε (z)| |x−y| ≤ kukW 1,∞ ·|x−y| .
0 z∈Ωε

53
Therefore, each uε is Lipschitz continuous with Lipschitz constant L = kukW 1,∞ . Taking the
limit as ε → 0 we obtain the result.
(ii) Consider the open set
.
Ω = {(x1 , x2 ) ; 1 < x21 + x22 < 4} \ {(x1 , x2 ) ; x2 = 0 , x1 > 0}.

Let u be the angle function, in polar coordinates. In other words,


.
q
u(x1 , x2 ) = θ , if (x1 , x2 ) = r(cos θ, sin θ) , r = x21 + x22 , 0 < θ < 2π .

6. (i) Consider first the case where Ω is a bounded, open, convex set. Let u be Lipschitz
continuous with constant C. For every fixed (n − 1)-tuple (x1 , x2 , . . . , xi−1 , xi+1 , . . . , xn ), the
function
s 7→ u(x1 , x2 , . . . , xi−1 , s, xi+1 , . . . , xn )
is defined for s in some open interval ]a, b[ (possibly empty). Moreover, it is Lipschitz continu-
ous of the same constant C. Being absolutely continuous, it is differentiable almost everywhere.
We thus conclude that the partial derivative
u(x + hei ) − u(x)
uxi (x) = lim .
h→0 h
exists for a.e. x ∈ Ω. An integration by parts shows that this function provides the weak
derivative Dxi u. Since u is bounded and |uxi (x)| ≤ C at every point x where the partial
derivative exists, it is clear that u ∈ W 1,∞ (Ω).
The result can be easily extended to a general open set Ω, observing that

kukW 1,∞ (Ω) = sup kukW 1,∞ (B(x,r)) ,


B(x,r)⊂Ω

where the supremum is taken over all open balls contained in Ω.


(ii) Consider any bounded open subset Ω0 ⊂ Ω. Then W 1,∞ (Ω0 ) ⊂ W 1,n+1 (Ω0 ). Hence by
Theorem 8.41 the function u is differentiable a.e. on Ω0 . By varying the set Ω0 we conclude
that u is differentiable a.e. on Ω.

7. A direct computation shows that


1
1 n
Z   
kf knLn (Ω) = cn · r n−1
ln ln 1 + dr < ∞
0 r
because the integrand on the right hand side is bounded. Moreover, for n ≥ 2 we have
 n
Z 1
1 1 1 
k∇f knLn (Ω) = cn · rn−1   · 1 · r 2 dr dr
0 1
ln 1 + r 1+ r

Z 1
1
≤ cn ·   dr < ∞ .
n
0 r ln 1 + 1r

54
8. (i) Let f be continuously differentiable. Then
Z
|f (0)| ≤ |f (x)| + |f 0 (y)| dy.
[0,x]

integrating both sides over the interval [−1, 1] we obtain


Z 1 Z 1
2|f (0)| ≤ |f (x)| dx + (1 − |x|)|f 0 (x)| dx ≤ kf kW 1,1 .
−1 −1

Since C 1 (Ω) is dense in W 1,1 (Ω), this functional can be extended by continuity to a bounded
linear functional on the whole space W 1,1 (Ω).
(ii) Next, assume Ω = B(0, 1) ⊂ R2 . For p > 2, setting γ = 1 − p2 , Morrey’s inequality yields

|f (0)| ≤ kf kC 0,γ (Ω) ≤ C · kf kW 1,p (Ω) ,

for some constant C and every f ∈ W 1,p ∩ C 1 . Hence the map T : f 7→ f (0) can be uniquely
extended to a bounded linear functional on the entire space W 1,p (Ω).
To see that this functional is not continuous on W 1,p (Ω) for p ∈ [1, 2], we prove that
|f (0)|
sup = ∞. (28)
f ∈W 1,2 (Ω) kf kW 1,2

Observing that, for every p ∈ [1, 2], one has kf kW 1,p ≤ Cp kf kW 1,2 , from (28) we conclude that
the functional f 7→ f (0) is unbounded in the space kf kW 1,p as well.
To prove (28), consider the function
.
 1 
f (x) = ln ln 1 +
|x|
and the decreasing sequence of continuous functions
 
. f (x)
fn (x) = min 1 , .
n

As shown in Problem 7, one has f ∈ W 1,2 (Ω). Hence


1
|fn (0)| = 1 , kfn kW 1,2 ≤ kf kW 1,2 → 0.
n

9. Assume p > 2 and set γ = 1 − p2 . Let f ∈ C0∞ (R3 ). Then for every t ∈ R the restriction of
f to the plane
.
Σt = {(t, x2 , x3 ) ; x2 , x3 ∈ R}
lies in Cc∞ (Σt ). Morrey’s inequality yields

|f (t, 0, 0)| ≤ kf kC 0,γ (Σt ) ≤ C kf kW 1,p (Σt ) .

we have Z  
|f (x̄1 , x2 , x3 )|p + |∇f (x̄1 , x2 , x3 )|p dx2 dx3 < ∞ .
R2

55
If p is suitably large, then f coincides a.e. with a Hölder continuous function on the plane
{(x1 , x2 , x3 ) ; x1 = x̄1 } .

10. Construct a sequence of smooth approximations uν ∈ Cc∞ (Rn ) such that kuν −ukW 1,p (Rn ) ≤ 2−ν−1
.
for every ν ≥ 1. Setting u0 = 0, vν = uν+1 − uν , we have

X
u = w0 + wν , kwν kW 1,p ≤ 2−ν forall ν ≥ 1 .
ν=1

We shall use the notation x = v + y, with v ∈ V , y ∈ V ⊥ . Observing that


Z nX ∞ Z X ∞ Z 
p p p p
(|wν | + |∇wν | ) dx = (|wν | + |∇wν | ) dv dy < ∞ ,
R ν=0 V ⊥ ν=0 y+V

by Fubini’s theorem we conclude that


Z X∞
(|wν |p + |∇wν |p ) dv < ∞ (29)
y+V ν=0

for a.e. z ∈ V ⊥ . If (29) holds, then the sequence of partial sums is absolutely convergent in
W 1,p (y + V ), hence also in C 0,γ (y + V ), by Morrey’s embedding theorem, with γ = 1 − m p.
Hence, restricted to the affine subspace y + V , the function u coincides a.e. with a Hölder
continuous function.

V

x = v+y
y y+V

V
0 v

Figure 2: Each affine subspace y + V has dimension m < p, hence Morrey’s inequality can be applied.

(ii) For any smooth function w ∈ Cc∞ (Rn ) with compact support, one has
Z Z Z
p
p
|w(y)| dy ≤ kwkC 0,γ (y+V ) dy ≤ C kwkpW 1,p (y+V ) dy = CkwkpW 1,p (Rn ) .
V⊥ V⊥ V⊥

By approximation, we conclude that kukLp (V ⊥ ) ≤ CkwkpW 1,p (Rn ) for every u ∈ W 1,p (Rn ).

11. Since the inequality


n
kgkC 0,γ (Ω) ≤ C kgkW 1,p (Ω) , γ =1−
p

is valid for every g ∈ Cc∞ (Ω), by approximation the same holds for every f ∈ W01,p (Ω), after a
modification on a set of measure zero.

56
12. Consider any open set Ω ⊂ Rn . Consider the open subsets
.
n o
Ω1/n = x ∈ Ω ; |x| < n , B(x, 1/n) ⊂ Ω . (30)

By the dominated convergence theorem, for any f ∈ Lp (Ω), 1 ≤ p < ∞, the approximations

. f (x) if x ∈ Ω1/n
fn (x) =
0 otherwise

converge to f in Lp . In turn, for ε < 1/n the mollifications Jε ∗ fn lie in Cc∞ (Ω) and converge
to fn as ε → 0. Hence f can be approximated in Lp (Ω) by smooth functions with compact
support.
Next, consider any sequence of functions fn ∈ Cc∞ (Ω). and assume kfn − f kL∞ → 0. This
implies that f is the uniform limit of a sequence of continuous functions which vanish on the
boundary of Ω. Hence f is a bounded continuous function which satisfies

lim sup {|f (x)| ; x ∈ Ω \ Ω1/n } = 0 . (31)


n→∞

Viceversa, every bounded continuous function f satisfying (31) lies in W00,∞ (Ω). Indeed, if
this holds, then we can approximate f with a sequence of functions fn,εn ∈ Cc∞ (Ω), choosing

1
fn,εn = Jεn ∗ fn = Jεn ∗ (f · χ ), εn << .
Ω1/n n

.
13. Let fk (x) = f (x)ϕ(k − |x|). Then fk is supported on the set where |x| ≤ k and coincides
with f for |x| ≤ k − 1. By the dominated convergence theorem
Z p !
∂f
kfk − f kpW 1,p ≤ C
X
|f |p +


∂xi dx → 0 as k → ∞.
|x|>k−1 i

Performing the mollifications fk,εk = Jεk ∗ fk with εk << 1/k we obtain a sequence of smooth
functions with compact support which converge to f in W 1,p (Rn ).

.
14. If u ∈ W 2,p (R+ ) then u is absolutely continuous, hence the limit u(0) == limx→0+ u(x)
is well defined. The even extension Eu(x) = u(|x|) is absolutely continuous, with derivative
(Eu)x (x) = (sign x) ux (|x|) for a.e. x ∈ R. Our assumptions imply
Z ∞  Z ∞ 
|Eu|p + |(Eu)x |p dx = 2 u|p + |ux |p dx < ∞.
−∞ 0

Hence Eu ∈ W 1,p (R).


In general, Eu ∈ / W 2,p (R). For example, take u(x) = x/2. Then Eu(x) = |x|/2 is a function
whose first derivative is (Eu)x = 21 sign x. However, the function x 7→ 21 sign x has a jump at
x = 0. Its distributional derivative is a Dirac distribution, concentrating a unit mass at x = 0.
Hence Eu does not have a weak second derivative.

57
15. (i) A direct computation yields
Z 1/q Z 1/q
−n
kuλ k
Lq = q
|u(λx)| dx = q
|u(y)| λ dy = λ−n/q kukLq .

Hence the conclusion holds with α = −n/q.


(ii) A similar computation yields
Z 1/p Z 1/p
k∇uλ kLp = |λ ∇u(λx)|p dx = |u(y)|p λp−n dy = λ(p−n)/p kukLp .

Hence the conclusion holds with β = (p − n)/p.


p−n
(iii) We have α = β if − nq = p . This is the case if 1
q = 1
p − n1 , i.e. if q = p∗ is the Sobolev
conjugate exponent to p.

16. Since H 1 (Ω) is a Hilbert space and the sequence (um )m≥1 is bounded, we can extract
a convergent subsequence umk * ũ in H 1 (Ω). Moreover, since the embedding H 1 (Ω) ⊂
L2 (Ω) is compact, by possibly extracting a further subsequence we obtain the convergence
kumk − ukL2 → 0. This clearly implies ũ = u, hence u ∈ H 1 (Ω).
To prove a bound on kukH 1 , set
.
M = lim inf kum kH 1 .
m→∞

By possibly extracting a subsequence and relabeling, we can assume

M = lim kum kH 1 .
m→∞

Writing
0 ≤ (um − u, um − u)H 1 = (um , um )H 1 + (u, u)H 1 − 2(um , u)H 1 ,
and letting m → ∞, we obtain

M 2 + kuk2H 1 − 2kuk2H 1 ≥ 0 .

Hence kukH 1 ≤ M .

17. (i) Consider the functions


n+1 n

. 1− n |x| if |x| ≤ n+1 ,
fn (x) =
0 otherwise.

Show that each fn has compact support in Ω. Moreover, kfn −f kW 1,p → 0. Choose a sequence
εn → 0 so that the mollified functions Jεn ∗ fn have compact support and converge to f in H 1 .
 
(ii) To show that f ∈ W01,2 (Ω0 ), let ϕ(x) = log log 1 + |x|1
. Notice that ϕ ∈ W 1,2 (Ω) as
claimed in problem 6. For each integer n, define the function
 o
n n+1
gn (x) = max 0 , min 1 − |x| , n − ϕ(x) .
n

58
Show that each gn has compact support in Ω0 . Moreover, kgn − f kW 1,p → 0. Choose a
sequence εn → 0 so that the mollified functions Jεn ∗ fn have compact support contained in
Ω0 and converge to f in H 1 .
(iii) If the functions ϕn ∈ Cc∞ (Ω0 ) form a Cauchy sequence w.r.t. the W 1,p norm, with p > 2,
then by Morrey’s inequality the sequence (ϕn )n≥1 converges to some continuous function g,
uniformly on the closed disc Ω. In particular, g = 0 at the origin. Derive a contradiction,
showing that f cannot coincide a.e. with a continuous function that vanishes at the origin.

18. (i) Since Ω is connected, by Corollary 8.16 the assumption implies that u coincides a.e. with
a constant. Since the set where u = 0 has positive measure, we conclude that u(x) = 0 for
a.e. x ∈ Ω.
(ii) If (8.82) fails, consider a sequence of functions un such that
1
kun kL2 = 1 , k∇un kL2 < ,
n

and meas {x ∈ Ω ; un (x) = 0} ≥ α for every n.
Since the embedding H 1 (Ω) ⊂ L2 (Ω) is compact, we can extract a subsequence (unk )k≥1 such
that kunk − ukL2 → 0 for some limit function u ∈ L2 (Ω), and moreover unk * u in H 1 (Ω).
By taking a further subsequence, we can assume the pointwise convergence unk (x) → u(x) for
a.e. x ∈ Ω.
By assumption, the weak gradients ∇unk converge to zero in L2 (Ω). Hence ∇u ≡ 0 and
u(x) = c for some constant c and a.e. x ∈ Ω. We claim that c = 0, because otherwise
Z Z
kunk − uk2L2 = |unk (x) − c|2 dx ≥ c2 dx ≥ αc2
Ω {x ; unk (x)=0}

for every k.
On the other hand, the strong convergence in L2 implies kukL2 = limk→∞ kunk kL2 = 1,
providing a contradiction.

19. Fix i ∈ {1, . . . , n} We claim that the function v = uxi , defined a.e. as the classical
derivative of u, provides the weak derivative of u on Rn .
In the following, without loss of generality we assume i = 1 and use the notation x = (x1 , x0 ).
Moreover, we consider the set
.
n o
N = x0 = (x2 , . . . , xn ) ∈ Rn−1 ; (t, x0 ) ∈ K for some t ∈ R .

By assumption, this set has zero (n − 1)-dimensional measure.


For any test function ϕ ∈ Cc∞ (Rn ), we now have
Z Z Z ∞ Z  Z ∞  Z
0 0
uϕx1 dx = uϕx1 dx1 dx = − ux1 ϕ dx1 dx = − ux1 ϕ dx .
Rn Rn−1 −∞ Rn−1 \N −∞ Rn

Hence ux1 = Dx1 u, proving our claim.

59
In turn, the assumptions u, ∇u ∈ Lp (Ω) imply u ∈ W 1,p (Ω).

20. (i) Take f (x) = g(x) = |x|−1/2


3
(iii) Take f (x) = g(x) = |x| 2 −n .

21. Assume 1 ≤ p < ∞. If the result were not true, we could find a sequence un ∈ W 1,p (Ω)
such that

kun kLp (Ω) = 1 , kun kLp (Ω0 ) → 0 , k∇un kLp (Ω) → 0 .

By the compact embedding W 1,p (Ω) ⊂⊂ Lp (Ω) there exists a subsequence unk such that
kunk − ukLp → 0 for some u ∈ Lp (Ω)
The above conditions imply kukLp (Ω0 ) = 0, k∇ukLp (Ω) = 0. Since Ω is connected, we conclude
that u ≡ 0. However, this is in contradiction with kukLp (Ω) = limn→∞ kukLp (Ω) = 1.

22. The set S can be represented as


n o
S = f : ]0, 1[ 7→ R ; kf kC 1 ≤ M , f 0 is Lipschitz continuous with constant M .

Assume kfn − f kC 0 → 0, with fn ∈ S for every n. By assumption, all functions fn , ∂x fn are


uniformly bounded and Lipschitz continuous with constant M . Hence they can be extended
by continuity to the closed interval [0, 1].
Since fn (x) → f (x) uniformly on [0, 1], this implies that f is also Lipschitz continuous with
constant M . By Ascoli’s theorem, by taking a subsequence we can assume the uniform con-
vergence ∂x fnk → v for some Lipschitz continuous function v. Together, the limits

fnk (u) → f (x) , ∂x fnk (x) → v(x) uniformly for x ∈ ]0, 1[

imply that v(x) = f 0 (x). Since this limit does not depend on the particular subsequence, we
conclude that the entire sequence ∂x fn converges to ∂x f (x), uniformly on ]0, 1[.
Notice that this implies that ∂x f is also Lipschitz continuous with constant M . The above
shows that f ∈ S, hence S is a closed subset of C 0 (]0, 1[). The above arguments prove both
(ii) and (i).

23. Let Eu ∈ W 1,p (Rn ) be an extension of u to the entire space Rn , with kEukW 1,p (Rn ) ≤
.
CkukW 1,p Ω) . Consider the mollifications uk = J1/k ∗ Eu.

24. The assumption implies that f is absolutely continuous on any bounded interval. Hence
f differentiable at a.e. point. Writing
Z x
f (x) = f (a) + g(s) ds,
a

we have Z x+1/n
1
gn (x) = g(s) ds .
n x

60
This implies the pointwise convergence gn (x) → g(x) at every Lebesgue point x of g, hence
almost everywhere.
To prove that kgn − gkL1 ([a,b]) → 0, let ε > 0 be given. Choose a continuous function ϕ such
that kg − ϕkL1 ([a, b+1]) < ε and define
Z x+1/n
1
ϕn (x) = ϕ(s) ds .
n x

Then
lim sup kgn − gkL1 ([a,b])
n→∞

≤ lim sup kgn − ϕn kL1 ([a,b]) + lim sup kϕn − ϕ|L1 ([a,b]) + lim sup kϕ − gkL1 ([a,b])
n→∞ n→∞ n→∞

≤ ε + 0 + ε.

Since ε > 0 was arbitrary, this proves the result.

25. The net smoothness of a function u ∈ W 2,2 (R3 ) is k − np = 2 − 32 = 12 . Therefore,


the Sobolev embedding theorem implies that, after a modification on a set of measure zero,
un ∈ C 0,1/2 , with kun kC 0,1/2 ≤ C, for some constant C and every n ≥ 1. Being the pointwise
limit of a sequence of uniformly Hölder continuous functions, u is Hölder continuous as well.

Chapter 9

 
1. The PDE can be rewritten as Lu = −f , where Lu = − (ux )x + (xux )y + (uy )y . To
prove that the operator L is uniformly elliptic, it suffices to check that the quadratic form
(ξ1 , ξ2 ) 7→ ξ12 + xξ1 ξ2 + ξ22 is strictly positive definite for (x, y) ∈ Ω. The conclusion is then
obtained by applying Theorem 9.8.

2. (i) The fact that B[u, v] = hu, vi♦ is a continuous bilinear map on H01 is clear. We need to
show that it is strictly positive definite. For |y| ≤ 1 one has
√ √ √
a2 + 2b2 + 2yab ≥ (2 − 2)(a2 + 2b2 ) + 2(a2 + 2b2 ) + 2yab ≥ (2 − 2)(a2 + 2b2 ) .

Therefore, by Poincare’s inequality,


Z √ Z
B[u, u] = 2 2
(ux + 2uy + 2yux uy ) dxdy ≥ (2 − 2) (u2x + u2y ) dxdy ≥ β kuk2H 1
Ω Ω

for some β > 0 and all u ∈ H01 (Ω).


(ii) We can now use the Lax-Milgram theorem and conclude that for every f ∈ L2 (Ω) there
exists a unique u ∈ H01 (Ω) such that B[u, v] = (f, v)L2 for every v ∈ H01 (Ω). By a formal

61
integration by parts, we find that this function u provides a weak solution to the elliptic
boundary value problem

−uxx − 2uyy − (2 + y)uxy = f on Ω ,
u = 0 on ∂Ω .

3. The operator L  is elliptic


 in a neighborhood of the point (x, y) ∈ R2 if and only if the
x 1
coefficient matrix is strictly positive definite. This is the case if and only if
1 y
.
(x, y) ∈ Ω+ = {(x, y) ∈ R2 ; x > 0, y > 0, xy > 1} .

The operator L is uniformly elliptic on a bounded domain Ω if and only if the closure Ω is
entirely contained in the open set Ω+ .

4. If φ ∈ H01 is a weak solution, then


Z Z
∇φ · ∇v dx = µ φ v dx for all v ∈ H01 (Ω).
Ω Ω

In particular, taking v = φ we obtain


Z Z
2
kφkL2 (Ω) = |∇φ| dx = µ |φ|2 dx = µkφk2L2 (Ω) .
2
Ω Ω

By assumption,
kφk2L2 (Ω)
β2 = sup ,
u∈H01 ,u6=0 k∇φk2L2 (Ω)

hence µ ≥ 1/β 2 .
(ii) Using the representation formula (9.57), the solution of the parabolic equation with initial
data u(0) = g is provided by

X
u(t) = e−µk t (g, φk )L2 φk , (32)
k=1

where φk ∈ H01 (Ω), the set {φk ; k ≥ 1} is an orthonormal basis of L2 (Ω) consisting of
eigenfunctions of the Laplace operator, and µk are the corresponding eigenvalues. Since µk ≥
β −2 for every k ≥ 1, from (32) it follows
∞ 2 ∞ 2
X −µk t X −t/β 2 2
ku(t)k2L2 = e (g, φk )L2 ≤ e (g, φk )L2 = e−2t/β kgk2L2 .

k=1 k=1

Taking square roots of both sides we obtain the result.

5. As in Theorem 9.9, let {φk ; k ≥ 1} be an orthonormal basis of L2 (Ω), consisting of


eigenfunctions of the operator −∆. For every v ∈ Cc∞ (Ω) we have
Z Z ∞
. 2
X
B[v, v] = |∇v| dx = − v · ∆v dx = µk (v, φk )2 ≥ µ1 kvk2L2 .
Ω Ω k=1

62
By continuity, the same inequality remains valid for every v ∈ H01 (Ω). On the other hand, the
eigenfunction φ1 ∈ H01 (Ω) satisfies
Z Z
2
B[φ1 , φ1 ] = |∇φ1 | dx = − φ1 · ∆φ1 dx = µ1 = µ1 kφ1 k2L2 .
Ω Ω

We thus obtain a representation for the first eigenvalue:


|∇v|2 dx
R
B[v, v]
µ1 = inf = inf RΩ . (33)
06=v∈H01 (Ω) kvk2L2 06=v∈H01 (Ω) 2
Ω |v| dx

Similarly,
|∇v|2 dx
R
B[v, v]
µ̃1 = inf = inf RΩe .
06=v∈H01 (Ω) kvk2L2 06=v∈H01 (Ω) Ω
2
e |v| dx

e implies H 1 (Ω) ⊂ H 1 (Ω).


The inclusion Ω ⊆ Ω e Hence the above representation formulas yield
0 0
µ̃1 ≤ µ1 .

.
6. Choose γ = maxt∈[0,T ] q(t) and consider the operator
.
Lγ u = − (p(t)u0 )0 + (γ − q(t))u .

Since p(t) ≥ θ > 0, the operator Lγ is uniformly elliptic. We claim that the bilinear form
Z T 
Bγ [u, v] = p(t)u0 (t)v 0 (t) + (γ − q(t))u(t)v(t) dt
0

is strictly positive definite on H01 (]0, T [). Indeed,


Z T
Bγ [u, u] ≥ p(t)|u0 (t)|2 dt ≥ βkukH 1
0

for some β > 0 and all u ∈ H01 (]0, T [).


As in the Theorem 9.9, the inverse operator L−1
γ is a linear, compact self-adjoint operator from
L2 (Ω) into itself. By the Hilbert-Schmidt theorem, the space L2 (Ω) admits an orthonormal
basis {φk ; k ≥ 1} consisting of eigenfunctions of L−1
γ . The corresponding eigenvalues satisfy
λk > 0, λk → 0 as k → ∞. This implies
 
1
− γ φk = Lφk .
λk
The above analysis shows that the eigenvalues of the Sturm-Liuville problem (9.85) are given
by µk = γ − λ1k . Hence limk→∞ µk = −∞.

7. By assumption, for each k ≥ 1 we have


Z Z
∇φk · ∇v dx = µk φk v dx
Ω Ω

for some eigenvalue µk and all v ∈ H01 (Ω). Assume j 6= k and choose v = φj . Then
Z Z
∇φk · ∇φj dx = µk φk φj dx = 0 .
Ω Ω

63
Hence (φk , φj )H 1 = 0.

8. (i) Let {ϕk ; k ≥ 1} be an orthonormal basis of L2 (Ω) consisting of eigenfunctions of


the Laplace operator ∆. We claim that, for every f ∈ L2 (Ω) and m ≥ 1, the linear system of
algebraic equations
m
X
B[um , ϕj ] = ck B[ϕk , ϕj ] = (f, ϕj )L2 j = 1, . . . , m
k=1

has a unique solution c1 , . . . , cm .


For this purpose, it suffices to show that the m × m matrix of coefficients A = (aij ), with
aij = B[ϕk , ϕj ], is invertible. Assume, on the contrary, that there exists a nonzero vector
ξ = (ξ1 , . . . , ξn ) such that
n
X
B[ϕk , ϕj ]ξj = 0 k = 1, . . . , m .
j=1
P
Then, setting v = j ξj ϕj 6= 0 we obtain
m
X
B[v, v] = B[ϕk , ϕj ]ξj ξk = 0,
j,k=1

contradicting the assumption that the bilinear form B[·, ·] is strictly positive definite.
(ii) If the identity
B[um , v] = (f, v)L2
holds for v = ϕ1 , . . . , ϕm , then by linearity it remains valid for every v ∈ span{ϕ1 , . . . , ϕm }.
In particular, since B is strictly positive definite, this implies

βkum k2H 1 ≤ B[um , um ] = (f, um )L2 ≤ kf kL2 kum kL2 ≤ kf kL2 kum kH 1 ,

for some constant β > 0. Therefore

kum kH 1 ≤ β −1 kf kL2 .

Thanks to the uniform bound, we can extract a subsequence (unj )j≥1 and achieve the weak
convergence unj * u in H01 (Ω). We now have

B[u, ϕk ] = lim B[um , ϕk ] = (f, ϕk ).


m→∞

By linearity
B[u, v] = lim B[um , v] = (f, v) (34)
m→∞

for every v ∈ span{ϕk ; k ≥ 1}. By approximation, (34) remains valid for every v ∈
span{ϕk ; k ≥ 1} = H01 (Ω). This proves that u is a weak solution to the elliptic problem.

9. According to (9.36), the solution u of the boundary value problem satisfies an abstract
equation of the form u − Ku = h, where K : L2 (Ω) 7→ L2 (Ω) is a compact operator and

64
h = L−1 −1
γ f , where Lγ is another compact operator. Moreover, we are assuming that the
operator I − K is a continuous bijection. By the open mapping theorem, the inverse operator
(I − K)−1 is continuous. Being the composition of a compact operator and a continuous one,
the map f 7→ u = (I − K)−1 L−1γ f is compact operator.

10. (i) By Problem 9 in Chapter 2, every function ϕ ∈ Cc∞ (Q) can be uniformly approximated
by a finite linear combination of the functions φm,n . Since Cc∞ (Q) is dense in L2 (Q), we
conclude that span{φm,n ; m, n ≥ 1} is dense in L2 (Q).
A straightforward computation shows that
 mπ 2  nπ 2
−∆φm,n = µm,n φm,n , µm,n = + .
a b

(ii) If µ1 is the smallest eigenvalue of −∆ on Ω, then by the result proved in problem 5. the
2 2
assumption Ω ⊆ Q implies µ1 ≥ µ1,1 = πa2 + πb2 . The conclusion now follows from (33).

11. This is a special case of problem 8. Consider the 2 × 2 matrix A = (aij ) and the vector
b = (b1 , b2 ) with entries
Z 3 Z 3
aij = B[ϕi , ϕj ] = ϕ0i (x)ϕ0j (x) dx , bi = 1 · ϕi dx .
0 0

An explicit calculation yields


   
2 1 1
A = , b = .
1 2 1

Solving the system of two algebraic equations


    
2 1 ξ1 1
= ,
1 2 ξ2 1

we find ξ1 = ξ2 = 1/3. The Galerkin approximation thus yields



x/3 if x ∈ [0, 1] ,
ϕ1 (x) + ϕ2 (x) 
u2 (x) = = 1/3 if x ∈ [1, 2] ,
3
(3 − x)/3 if x ∈ [2, 3] .

The exact solution of the boundary value problem is u(x) = 32 x − 12 x2 .

12. (i) Here L is the symmetric operator defined by


1 1
Lu = −(2ux )x − (yuy )x − (yux )y − (3uy )y .
2 2
The corresponding symmetric bilinear form is
Z 
y 
B[u, v] = 2ux vx + (ux vy + uy vx ) + 3uy vy dx .
Ω 2

65
On the open domain Ω = {(x, y) ; x2 + y 2 < 1}, the operator L is uniformly elliptic. Indeed,
for |y| ≤ 1 we have

2ξ12 + 2yξ1 ξ2 + 3ξ22 ≥ ξ12 + ξ22 for every ξ = (ξ1 , ξ2 ) ∈ R2 .

(ii) Define the energy as


Z Z 
1 1 1 1 
E(t) = kut k2L2 (Ω) + B[u, u] = u2t dx + 2u2x + yux uy + 3u2y dx .
2 2 2 Ω 2 Ω

Differentiating w.r.t. time, and using the boundary condition u = ut = 0 on ∂Ω, performing
an integration by parts we obtain
d
E(t) = (ut , utt )L2 + B[u, ut ]
dt
Z Z 
y 
= ut utt dx + 2ux uxt + (uy uxt + ux uyt ) + 3uy uyt dx
Ω Ω 2
Z Z  ux 
= ut utt dx − ut 2uxx + yuxy + 2uyy + dx = 0 .
Ω Ω 2

13. (i) The fact that on H01 (Ω) the norm k·k♦ is equivalent to the H 1 norm is a straightforward
consequence of the fact that the bilinear form B[·, ·] is strictly positive definite.
(ii) Repeat the arguments used in the proof of Lemma 9.7.

14. For any g ∈ L2 (Ω) the formula



X
St g = e−µk t (g, φk )L2 φk . (35)
k=1

defines a trajectory t 7→ St g ∈ L2 (Ω). Here {φk ; k ≥ 1} is an orthonormal basis of L2 (Ω).


We need to show that this map is n times continuously differentiable, for every n ≥ 1. Toward
this goal we observe that, for a given N ≥ 1, the partial sum is continuously differentiable:
N N
!
dn X −µk t X
e (g, φ ) 2
k L kφ = (−µk )n e−µk t (g, φk )L2 φk .
dtn
k=1 k=1

We claim that, as N → ∞, the above sum converges to a well defined limit, uniformly for t
in compact subsets of ]0, ∞[ . Since the φk form an orthonormal sequence, it suffices to prove
that X 2
lim (−µk )n e−µk t (g, φk )L2 = 0

N →∞
n>N

uniformly for t in compact subsets of ]0, ∞[ .


Consider any interval [a, b], with 0 < a < b < ∞. Then
.
M = sup µn e−µt < ∞ .
µ≥0,t∈[a,b]

66
Given any ε > 0, choose N so large that
X
M |(g, φk )|2 < ε .
n>N

2 = kgk2L2 < ∞. Then, for every t ∈ [a, b],


P
This is certainly possible because n≥1 |(g, φk )|

X 2 X
n −µk t
(−µk ) e (g, φk )L2 ≤ M |(g, φk )|2 < ε .

n>N n>N

15. For γ > 0, the bilinear form


Z
Bγ [u, v] = ∇u · ∇v + γuv dx

satisfies
.
Bγ [u, u] ≥ β kuk2H 1 , β = min{1, γ}.
Hence for every f ∈ L2 (Ω) there exists a unique u ∈ H 1 (Ω) such that

Bγ [u, v] = (f, v)L2 for every v ∈ H 1 (Ω).

The map f 7→ u = L−1 2


γ f is a self-adjoint, compact linear operator from L (Ω) into itself.

A function u ∈ L2 (Ω) is a weak solution to the Neumann problem if and only if

u − L−1 −1
γ γu = Lγ f .

This can be written in the abstract form


. .
(I − K)u = h , K = γL−1
γ , h = L−1
γ f. (36)

The equation (36) has a solution if and only if

h ∈ Range(I − K) = [Ker(I − K)]⊥ .

This is the case if and only if (h, v)L2 = 0 for every solution v of v − Kv = 0, i.e. for every
weak solution of the homogeneous Neumann problem

−∆u = 0

 on Ω ,

(37)
 ∂u = 0

on ∂Ω .
∂ν
By definition, v ∈ H 1 (Ω) is a weak solution of the above problem if
Z
∇v · ∇w dx = 0 for every w ∈ H 1 (Ω) .

Choosing w = v, this implies Z


|∇v|2 dx = 0 .

67
Since the open set Ω is connected, we conclude that the solutions of (37) are precisely the
constant functions.
Going back to the original problem,Rby the previous analysis the Neumann boundary value
problem has a solution ifRand only if Ω f · v dx = 0 for every constant function v. Of course,
this holds if and only if Ω f dx = 0.

16. On the space H02 (Ω) consider the continuous bilinear form
Z
.
B[u, v] = ∆u ∆v dx .

We claim that there exists β > 0 such that B[u, u] ≥ β kuk2H 2 , namely
Z X  X  Z X Z X Z
uxi xi uxj xj dx ≥ |uxi xj |2 dx + |uxi |2 dx + |u|2 dx . (38)
Ω i j Ω ij Ω i Ω

If u ∈ Cc∞ (Ω), integrating twice by parts we obtain


Z Z Z
uxi xi uxj xj dx = − uxi xi xj uxj dx = uxi xj uxi xj dx .
Ω Ω Ω

Hence Z X  X  Z X
uxi xi uxj xj dx = |uxi xj |2 dx . (39)
Ω i j Ω ij

Applying Poincare’s inequality to each function uxi we obtain


Z Z Z X
|uxi |2 dx ≤ C · |∇uxi |2 dx = C · |uxi xj |2 dx (40)
Ω Ω Ω j

for some constant C. Similarly,


Z Z X
2
|u| dx ≤ C · |uxi |2 dx . (41)
Ω Ω j

Together, (39)–(41) yield (38), whenever u ∈ Cc∞ (Ω). By an approximation argument, the
same estimate is valid for every u ∈ H02 (Ω).
Since B[·, ·] is strictly positive definite, the Lax-Milgram theorem yields the existence of a
unique u ∈ H02 (Ω) such that
B[u, v] = (f, v)L2 for every v ∈ H02 (Ω) .
By definition, u is the unique weak solution of the boundary value problem
∆2 u x ∈ Ω,

 = f

 u = ∂u

= 0 x ∈ ∂Ω ,
∂ν

Chapter 10

68
1. Since K is compact, it can be covered by finitely many of the sets Ai , say K ⊆ A1 ∪ A2 ∪
· · ·∪An . Consider the functions ρi (x) = d(x, Aci ), where Aci = K \Ai denotes the complement
of the set Ai . These functions are all Lipschitz continuous. Namely |ρi (x) − ρi (y)| ≤ d(x, y).
Hence the function r(x) = maxi ρi (x) is also Lipschitz continuous.
We now observe that, if x ∈ Ai then ρ(x) ≥ ρi (x) > 0. Hence the minimum of the continuous
function r(·) on the compact set K is strictly positive. By construction, it is now clear that
.
the constant ρ = minx∈K r(x) > 0 satisfies the requirement.

2. (i) If xn → x̄ then every subsequence converges to x̄ as well. To prove the converse


implication, assume that the sequence does not converge to x̄. Hence there exists ε > 0
such that for every N one can find n > N such that d(xn , x̄) > ε. If this holds, then we
can construct a subsequence (xnk )k≥1 such that d(xnk , x̄) > ε for every k. Clearly, from this
subsequence one cannot extract any further subsequence converging to x̄.
(ii) If a convergent subsequence (xnk )k≥1 exists, then it would be Cauchy. In particular,
limj,k→∞ d(xnj , xnk ) = 0. but this is impossible if d(xnj , xnk ) ≥ δ for j 6= k.
(iii) The assumption implies that, given ε > 0, from any subsequence (xn )n∈I one can extract
a further subsequence (xn )n∈I 0 such that

d(xm , xn ) < 2ε for all m, n ∈ I 0 .

Here I 0 ⊂ I ⊂ N are infinite sets of natural numbers.


We argue by induction on k = 1, 2, . . . Let I1 ⊂ N be a infinite set of indices such that

d(xm , xn ) < 2−1 for all m, n ∈ I1 .

After Ik−1 ⊂ N has been constructed, we choose an infinite set Ik ⊂ Ik−1 such that

d(xm , xn ) < 2−k for all m, n ∈ Ik .

After all the sets Ik have been constructed, we choose a sequence n1 < n2 < n3 < · · · such
that nk ∈ Ik for every k. We claim that the sequence (xnk )k≥1 is Cauchy. Indeed, if m < n,
then d(xm , xn ) ≤ 2−m . Since the space E is complete, this subsequence has a limit.

3. This is an elementary integral:


Z 1/2 Z − ln 2
1 1 1
kf kL1 = dx = dy = .
0 x(ln x)2 −∞ y 2 ln 2

Next, observe that the mollifier Jε is supported on the interval −ε, ε] and strictly positive on
the subinterval [−ε/2, ε/2]. Namely, it satisfies

δ ε

 if 0 < x < ,
.

Jε (x) ≥ φε (x) = ε 2


0 otherwise ,

69
for some constant δ > 0. For 0 < x < 1/6, choosing ε = 2x we obtain
Z 2x Z ln 2x
δ 1 δ 1 δ
F (x) ≥ (J2x ∗f )(x) ≥ (φ2x ∗f )(x) = 2
ds = 2
dy = .
0 2x s(ln s) 2x −∞ y 2x| ln(2x)|

/ L1 (R).
Hence F ∈

4. All functions fn are uniformly bounded, because

|fn (x)| ≤ |fn (0)| + kgkL1 for all n ≥ 1, x ∈ R

and by assumption the values fn (0) range in a bounded set.


Using the previous Problem 2 (iii), given ε > 0 it suffices to prove that from any subsequence
one can extract a further subsequence (fnk )k≥1 such that

lim sup kfnj − fnk kC(R) ≤ ε . (42)


j,k→∞

Toward this goal, choose a constant M large enough so that


Z −M Z ∞
ε
g(x) dx + g(x) dx < .
−∞ M 2

We claim that, on the compact interval [−M, M ], the functions fn are equicontinuous. Indeed,
R x+δ
Fix x ∈ [−M, M ] and  > 0. Choose δ > 0 such that x−δ g(s) ds < . Then |fn (y)−fn (x)| < 
for all y ∈ [x − δ , x + δ] and n ≥ 1.
Given any subsequence, we can thus apply Ascoli’s theorem and extract a further subsequence
(fnk )k≥1 which converges uniformly on the interval [−M, M ]. We now estimate (42) by writing
n
lim sup kfnj − fnk kC(R) = max lim sup kfnj − fnk kC(]−∞,−M ])
j,k→∞ j,k→∞
(43)
+ lim sup kfnj − fnk kC([−M,M ]) + lim sup kfnj − fnk kC([−M,M ])
j,k→∞ j,k→∞

We now have
lim sup kfnj − fnk kC([−M,M ]) = 0 .
j,k→∞

Moreover
Z −M
lim sup kfnj − fnk kC(]−∞,−M ]) ≤ lim sup |fnj (−M ) − fnk (−M )| + 2 g(x) dx ≤ 0 + ε ,
j,k→∞ j,k→∞ −∞
Z ∞
lim sup kfnj − fnk kC([M,∞[) ≤ lim sup |fnj (M ) − fnk (M )| + 2 g(x) dx ≤ 0 + ε .
j,k→∞ j,k→∞ M

Hence (42) holds.

.
5. Set gn (x) = arctan fn (x) and consider the functions
. .
a(x) = lim inf gn (x) ≤ lim sup gn (x) = b(x) .
n→∞ n→∞

70
.
The functions a, b are Lebesgue measurable, and so is the set A = {x ∈ R ; − π < a(x) =
.
b(x) < π}. By replacing each function fn with f˜n = χA · fn , it is not restrictive to assume
that the sequence (fn )n≥1 converges pointwise at every point x ∈ R. By Fatou’s lemma,
Z   Z
kf kL1 (R) = lim |fn (x)| dx ≤ lim inf |fn (x)| dx ≤ C .
n→∞ n→∞

6. Let f : [a, b] 7→ R be absolutely continuous, and let A ⊂ [a, b] be a set with measure zero.
Fix any ε > 0. We need to show that meas(f (A)) ≤ ε.
Toward this goal, let δ > 0 be such that N
P
i=1 |f (bi ) − f (ai )| < P
ε for any finite family of
disjoint intervals [ai , bi ] ⊂ [a, b], i = 1, . . . , N , with total length ni=1 (bi − ai ) < δ. Since
meas(A) = 0, there exists an open set V ⊇ A such that meas(V ) < δ. Since V ⊂ R is open,
we can write V as S a countable union of its connected components, which are disjoint open
intervals: V = k≥1 ]ck , dk [ . We now have

n
X   n
X
meas(f (A)) ≤ meas(f (V )) ≤ sup meas f ([ck , dk ]) = sup f (Mk )−f (mk ) . (44)

n n
k=1 k=1

Here mk , Mk ∈ [ck , dk ] are points where the continuous function f attains respectively its
minimum and maximum, restricted to the interval [ck , dk ]. Observing that
n
X n
X
|mk − Mk | ≤ (dk − ck ) < δ for all n ≥ 1,
k=1 k=1

we conclude that the right hand side of (44) is ≤ ε.

7. (i) Choose a subsequence such that kfnk kL1 ≤ 2−k for every k ≥ 1. Then for any N ≥ 1
we have Z 1 Z 1X

sup |fn (x)| dx ≤ |fn (x)| dx ≤ 2−N .
0 n>N 0 n>N

Calling
.
n o
Aε = x ∈ [0, 1] ; lim sup |fn (x)| ≥ ε ,
n→∞

for every integer N we have


1
2−N
  Z 
meas(Aε ) ≤ meas {x ∈ [0, 1] ; sup |fn (x)| ≥ ε} ≤ sup |fn (x)| dx ≤ .
n>N 0 n>N ε

Since N is arbitrary, this yields meas(Aε ) = 0. The above shows that lim supn→∞ |fn (x)| = 0
for a.e. x ∈ [0, 1]. Hence limn→∞ fn (x) = 0 for a.e. x.
(ii) For 2k < n ≤ 2k+1 , define

n − 2k − 1 n − 2k
  
 1 if x∈ , ,


2k 2k
fn (x) =


0 otherwise .

71
8. Let B(xj , rj ), j ≥ 1 be a countable family of disjoint open balls contained in Ω. For each
j, let gj be the characteristic function of the set B(xj , rj ). Clearly, all these functions lie in
Lp (Ω) and are linearly independent, hence the space Lp (Ω) is infinite dimensional.
Moreover, kgj kL∞ = 1, kgi − gj kL∞ = 1 for i 6= j.
To cover the case 1 ≤ p < ∞, define
 1/p
. 1
fj (x) = cj gj (x) , cj = .
meas(B(xj , rj ))
Then
kfj kLp (Ω) = 1 , kfi − fj kLp (Ω) = 21/p for all i 6= j .

9. It is clear that the set S is totally ordered: if (t1 , f (t1 )) and (t2 , f (t2 )) lie in S, with t1 ≤ t2 ,
then (t1 , f (t1 ))  (t2 , f (t2 )). To prove that S is maximal, assume (x, y) ∈ / S. To fix the ideas,
assume f (x) < y, the other case being similar. Then there exists t > x such that f (t) < y.
The set S ∪ {(x, y)} is not totally ordered, because the two relations
(x, y)  (t, f (t)), (t, f (t))  (x, y)
are both false.
A maximal totally ordered subset of different kind is
.
S = {(−1, y) ; y ≤ 0} ∪ {(x, 0) ; x ∈ [−1, 1]} ∪ {(1, y) ; y ≥ 0} .

10. The proof is by induction. When m = 2, this is the standard Hölder’s inequality. Assume
that the result is true for m = 2, 3, . . . , N − 1. Let fk ∈ Lpk (Ω), with p11 + p12 + . . . + p1N = 1.
If pN = ∞, then by the inductive hypothesis we immediately get
Z Z N
Y
|f1 · · · fN −1 · fN | dx ≤ |f1 · · · fN −1 | dx · kfN kL∞ ≤ kfk kLpk .
Ω Ω k=1

If 1 ≤ pN < ∞, choose q so that


1 . 1 1 1 1
= 1− = + + ... + .
q pN p1 p2 pN −1
The standard Hölder inequality yields
Z
|f1 · · · fN −1 · fN | dx ≤ kf1 · · · fN −1 kLq kfN kLpN . (45)

We now observe that pq1 + · · · + q


pN = 1, while |fk |q ∈ Lpk /q (Ω) for k = 1, . . . , N − 1. The
inductive assumption yields
Z 1/q N −1
!1/q N −1
kfkq kLpk /q
Y Y
kf1 · · · fN −1 kLq = |f1 |q · · · |fN −1 |q dx ≤ = kfk kLpk .
Ω k=1 k=1
(46)
Together, (45)-(46) yield the result for m = N . By induction, the generalized Hölder inequality
is valid for every m ≥ 2.

72

You might also like